General Surgery Flashcards

1
Q
  1. How would you go about investigating a 60 year old lady who presented to you having found a lump, about 2cm across, in her left breast?
A
  1. History
  2. Examination
  3. Imaging
  4. Biopsy (FNA/Core)
How well did you know this?
1
Not at all
2
3
4
5
Perfectly
2
Q

63.Key points of history taking re: lump in breast tissue? (9)

A
  • Lump location, size, changes, first noticed, tender, associated skin changes
  • Hx of breast lumps, cysts, fibroadenomas,
  • Menstrual hx,
  • Medications, including the contraceptive pill and HRT
  • Fx hx, including relatives affected by cancers
  • Ethnic background
  • Risks that suggest genetic susceptibility, such as family history, male breast cancer.
  • Trauma – accidental/iatrogenic
  • Systemic symptoms
How well did you know this?
1
Not at all
2
3
4
5
Perfectly
3
Q
  1. Examination technique re: lump in breast tissue
A

Inspection

  • Hands by side and above head
  • Breast contour
  • Skin changes: erythema, dimpling, puckering, peau d’orange
  • Nipple changes: inversion, distortion, eczema, nodules, ulders, discharge

Palpation

  • Four quadrants with a systematic approach
  • Nipple
  • Axillary tail
  • Lymph nodes
  • Axillary and supraclavicular
How well did you know this?
1
Not at all
2
3
4
5
Perfectly
4
Q

Imaging requests re: lump in breast tissue (5)

A
  • Mammography
    • Investigation of choice if >30yrs
    • USS should always accompany to increase accuracy -
  • US <30yrs
  • MRI
    • Is only funded under medicare for high-risk women (e.g. BRCA1/BRCA2 mutation carrier
  • CT (preoperative staging)
  • Bone scan (preoperative staging)
How well did you know this?
1
Not at all
2
3
4
5
Perfectly
5
Q

Algorithm for managing a breast symptom or screening abnormality

A
How well did you know this?
1
Not at all
2
3
4
5
Perfectly
6
Q

What are the risk factors for breast cancer? (11)

A
  • increasing age
  • family hx (1st>2nd degree relatives, onset, bilaterality, BRCA1/2 (autosomal dominant)
  • previous hx of breast cancer or carcinoma in-situ
  • early age of menarche (<12 years)
  • late age of menopause (age >55years)
  • late age at first full time pregnancy (age <20 years protective)
  • nulliparity
  • previous breast biopsies showing non-malignant abn. (ductal carcinoma insitu, lobar carcinoma in situ, atypical ductal hyperplasia
  • hormal therapy - OCP, HRT
  • Radiation at a young age
  • physical activity (reduces circulation of oestrogren)
  • chronic alcohol intake
How well did you know this?
1
Not at all
2
3
4
5
Perfectly
7
Q

List the options for adjuvant treatment for breast cancer (5)

A
  • radiotherapy
  • hormone therapy (tamoxifen, aromatase inhibitors)
  • chemotherapy (Anthracyclines, taxanes)
  • adjuvant anti-HER2 treatment (Trastuzumab - Herceptin)
  • Others (Aspirin, Bisphosphonates,Neoadjuvant treatment, lifestyle, radiotherapy)
How well did you know this?
1
Not at all
2
3
4
5
Perfectly
8
Q

Define Adjuvant Therapy

A

Aims at adding to definitive treatment to reduce the risk of both local recurrence and distant metastases in patients in whom there is a high risk of occult disease (micrometastases)

How well did you know this?
1
Not at all
2
3
4
5
Perfectly
9
Q

Describe Tamoxifen (Hormone Therapy) in Breast Cancer

A
  • MOA
    • Blocks oestrogen receptor inside the breast cancer cell. This stops oestrogen making the breast cancer cell grow.
  • Course
    • 1 daily tablet, 5 year course after other treatments
  • Benefits
    • Reduces risk of early breast cancer, new breast cancer developing. Can be used with other treatments for breast cancer like radiotherapy and chemotherapy
    • Strengthens bones rather than reducing bone density (unlike aromatase inhibitors can)
    • May lower cholesterol and reduce the risk of developing heart disease
    • Works either before or after menopause.
    • Agonist at bones and endometrial tissue (endometrial cancer)
  • Side effects
    • Does not cause menopause but side effects may be similar
    • Common
      • Hot flushes and sweats
      • Irregular vaginal bleeding in women who have not been through menopause
      • Vaginal irritation, dryness or discharge
      • Fluid retention and weight gain
    • rare
      • DVT, PE (similar risk to OCP)
    • Fertility
      • Do not become pregnant or breast feed for 1-2 months after stopping.
      • Can increase fertility so must use barrier contraception (do not take OCP with tamoxifen)
    • Can interact with warfarin
How well did you know this?
1
Not at all
2
3
4
5
Perfectly
10
Q

Describe Aromatase Inhibitors (Hormone Therapy) in Breast Cancer

A
  • First line and only work in post-menopausal women with PR or ER positive cancers.
    • Cannot use in pre-menopausal as it muffs up negative feedback to pituitary and makes things worse
  • MOA
    • Blocks aromatase, which helps make oestrogen in body tissues such as muscle, fat and the adrenal glands.
  • course
    • Tablets taken once daily
    • Usual treatment course 5 years.
    • Anastrazole, letrozole, exemestane
  • Benefits
    • Reduce risk of early breast cancer coming back, reduce risk of new breast cancer developing
    • Can be used with radio/chemotherapy
    • Can be substituted for tamoxifen in post-menopausal women
    • Less likely to cause DVT/PE or cancer of the endometrium than tamoxifen.
  • Side effects
    • Common
      • Muscle aches and pains
      • Hot flushes
      • Vaginal dryness
  • Contraindications
    • Osteoporosis
  • Follow-ups
    • Check Vit D levels, take daily calcium and Vit D supplements
    • Yearly bone scan
How well did you know this?
1
Not at all
2
3
4
5
Perfectly
11
Q

Describe Tratsuzumab (Adjuvant anti-HER2 treatment) in Breast Cancer

A
  • monoclonal antibody directed against the extracellular domain of human growth factor receptor 2 (HER2), a tyrosine kinase involved in cell growth and proliferation
  • prognosis
    • HER2 pos à worse prognosis
  • Clinical trials:
    • use of trastuzumab with chemotherapy improves disease-free and overall survival.
  • Side effects
    • Main risk is cardiac toxicity (up to 4%), hence baseline and surveillance echocardiograms are required.
  • Contraindicated
    • Anthracycline (due to cardiac toxicity)
How well did you know this?
1
Not at all
2
3
4
5
Perfectly
12
Q

Describe other adjuvant Therapies for Breast Cancer

A
  • Aspirin
    • Recent data from a large population cohort study suggests aspirin has a role in prevention of recurrence, although no randomised data exists and optimal dose is undefined.
  • Bisphosphonates
    • Six-monthly IV, or weekly orally,
    • Lowers cancer recurrence (both bone and other organs)
    • Direct anti-tumour effect is in keeping with that seen in metastatic breast cancer
  • Neoadjuvant treatment
    • Chemotherapy, hormone therapy, trastzumab may be considered for large or locally advanced tumours.
  • Lifestyle factors
    • Healthy diet
    • Regular exercise
    • Healthy weight range
  • Radiotherapy
    • After surgery
    • Down staging for neoadjuvant
  • supportive care
    • cancer-centre psychologists
    • information packs through National Breast and Ovarian cancer centre and cancer council websites
    • support groups
How well did you know this?
1
Not at all
2
3
4
5
Perfectly
13
Q
  1. When is systemic therapy used in breast cancer?
A
How well did you know this?
1
Not at all
2
3
4
5
Perfectly
14
Q
  1. What are the surgical options for a small confirmed cancer which is sited laterally in the breast, is not fixed to the skin, deep tissue or nipple? what are the advantages and disadvantages of the options?
A
  • Mastectomy
    • Radical (Halsted) - removal of breast, axillary lymph nodes and both pectoralis muscles - No longer indicated
    • Modified Radical (Patey) Mastectomy: Similar to radical but with preservation of pectoralis major
    • simple mastectomy: removal of the breast with no dissection of the axilla, except for the region of axillary tail, which usually has a few nodes attached to it in the anterior group - standard for invasive breast cancer
  • Breast Conserving Surgery (“Lumpectomy”)
    • Wide local excision: remove tumour with 1cm macroscopic margin of normal breast tissue.
    • quadrantecomy: removal of an anatomic quadrant
How well did you know this?
1
Not at all
2
3
4
5
Perfectly
15
Q
  1. what are the advantages and disadvantages of mastectomy?
A
  • advantages
    • No post-op RT needed unless high risk cancer
    • Lower rates of local recurrence
    • Can have reconstructive surgery
    • No follow-up imaging needed for that side
    • Psychologically better re: recurrence fears
    • Better for small breasts
  • disadvantages
    • Longer operation
    • Poorer cosmesis
    • Psychological impact on appearance and sexual dysfunction
How well did you know this?
1
Not at all
2
3
4
5
Perfectly
16
Q
  1. what are the advantages and disadvantages of lumpectomy
A
  • advantages
    • Similar survival benefits
    • Preservation of breast shape and skin
    • Shorter operation
    • Psychological advantage for appearance
  • disadvantages
    • Needs post-op RT
    • Higher rates of local recurrence
    • May need re-operation for margins
    • Quadrantectomy has poor cosmetic outcome
How well did you know this?
1
Not at all
2
3
4
5
Perfectly
17
Q
  1. What are important differential diagnoses for a breast lump?
A
  • Common
    • Fibroadenoma
    • Fibrocystic breast
    • Fat necrosis
    • Intraductal papilloma
    • Breast abscess
    • Atypical ductal hyperplasia (ADH) and atypical lobular hyperplasia (ALH)
    • Invasive breast cancer
    • Ductal carcinoma in-situ
  • Uncommon
    • Phyllodes tumor
    • Adenoma
    • Radial scar
    • Lobar carcinoma in situ (LCIS)
How well did you know this?
1
Not at all
2
3
4
5
Perfectly
18
Q
  1. What is the influence of a patients age on the choice of imaging for breast cancer?
A

In women >30 years breast lump should have diagnostic mammogram plus ultrasound to increase diagnostic accuracy, better characterize lesions and identify the presence of satellite lesions.

  • Mammography has a high false positive rate in young patients

In women <30 years – USS is the investigation of choice because the presence of denser breast tissue make evaluation with mammography less useful. Another consideration is the radiation to the breast tissue of younger women if mammography is performed.

  • USS Is most useful for Cystic lesions

MRI is the most sensitive investigation for breast cancer and the non-ionising nature makes it ideal for younger patients, however it is expensive and relatively less available and therefore not largely used.

How well did you know this?
1
Not at all
2
3
4
5
Perfectly
19
Q
  1. A 26 year old woman presents to the ED with a painful, hot swollen left breast. she is 5 week spost partum with her first child and has been breast feeding successfully. She was prescribed anti-biotics for the problem 5 days ago by her GP but has been getting worse. she has no systemic previous medical hx, and takes no regular medications.
A

Check for sepsis:

  • SIRS criteria:
    • HR > 90bpm
    • Temperature > 38˚C or < 36 ˚C
    • RR > 20 or PaO2 < 32mmHg (< 4.2kPa)
    • WCC > 12 or < 4

Lactational mastitis: usually occurs in first 3 months of breast feeding

  • Risk factors;
    • Cracked nipple
    • Poor feeding technique
  • Organism
    • Staph. Aureus
  • Typical presentation
    • Pain, swelling, erythema
  • Tx
    • Simple analgesia: paracetamol or NSAIDs
    • Empirical antibiotics (flucloxacillin)
    • Continued feeding or milk expression
    • If septic, IV antibiotics
    • If abscess develops, should be drained with repeated US-guided aspiration until resolution.
    • Surgery only if skin is threatened or necrotic
How well did you know this?
1
Not at all
2
3
4
5
Perfectly
20
Q
  1. How would you go about investigating a 16 year old girl who presented to you having found a lump, about 2cm across, in her left breast?
A

History:

  • Duration – when and how was the lump first noticed
  • Change – has the lump gotten bigger/ smaller, does it change with the menstrual cycle
  • Trauma – accidental or iatrogenic breast trauma
  • Pain – is the lump painful
  • Skin changes – erythema, skin puckering
  • Nipple changes – inversion, distortion, discharge
  • Risk factors – previous breast lesions, FHx of breast cancer, menarche, menstrual history and OCP use, smoking, alcohol, pregnant?
  • Systemic symptoms – weight loss, fever, night sweats, back pain, neurological changes, SOB, cough etc.

Inspection and examination with chaperone present

Document details of any findings – size, shape, consistency, mobility, tenderness, fixation and exact position

Management ‘

  • Benign breast lumps are very common in this age group, so it’s important to provide reassurance while ruling out breast cancer.
  • An appropriate management strategy can be to watch and wait to see if the lump resolves over the period of 2-3 menstrual cycles
    • If lump has not resolved then USS should be performed.

Most likely fibroadenoma

  • Localized form of ANDI (abnormalities of normal development and involution).
  • Smooth, firm, highly mobile
  • Larger fibroadenomas should be distinguished from benign phyllodes tumours.
How well did you know this?
1
Not at all
2
3
4
5
Perfectly
21
Q
  1. A 30 yao woman presented with a painful lump in her left breast 5 days after childbirth. This is a photograph of the breast. What do you see?
A

Describe lesion:

  • Location on a clock face or quadrant
  • Size and symmetry when compared to the other breast
  • Erythema
  • Purulence

Likely diagnosis: (in picture)

How well did you know this?
1
Not at all
2
3
4
5
Perfectly
22
Q
  1. What is the most likely organism that might be causing acute, lactation associated, breast abscess?
A
  • Commensal skin bacterium. Almost always Staphylococcus Aureus.
    • Enters through lactiferous duct or nipple trauma (cracks, fissures).
How well did you know this?
1
Not at all
2
3
4
5
Perfectly
23
Q
  1. What is the management of an acute lactation associated breast abscess
A
  • Mastitis
    • Rf: cracked nipple, poor feeding technique causing milk stasis
    • Flucloxacillin 500mg PO, 6hr x 5 days
    • Continued feeding and milk expression
    • Paracetamol or ibuprofen for pain control
      • In persistent cases: midstream milk sample MCS, and consider treating for MRSA.
    • US, and biopsy if palpable mass remains after the infection is cleared.
  • Abscess
    • Flucloxacillin
    • Refer to surgeon for needle aspiration to drain abscess
      • Incision and drainage if not effected.
    • Can occur with mastitis except a fluctuant mass is palpable
How well did you know this?
1
Not at all
2
3
4
5
Perfectly
24
Q
  1. a 34 yoa woman presented with a painful lump in her left breast 5 days after childbirth. This is a photograph of the breast. What is the likely diagnosis and what is the management?
A

Describe the lesion:

  • Location on a clock face or quadrant
  • Size and symmetry when compared to the other breast
  • Erythema
  • Purulence

Likely diagnosis – Lactation mastitis +/- breast abscess

DDx: Galactocele (usually non-painful), Inflammatory breast cancer

Management:

  • Continued breast feeding/ expression of milk
  • Simple analgesia
  • Education and reassurance
  • Antibiotics (flucloxacillin)
  • Aspiration with USS guidance or surgical incision and drainage – milk and pus aspirated should be sent for MCS
How well did you know this?
1
Not at all
2
3
4
5
Perfectly
25
Q
  1. A 50 year old woman is diagnosed with breast cancer. Her surgeon recommends a “lumpectomy” and sentinel node biopsy. The pathology report shows a 20mm grade 3 ER positive, PR positive, HER-2 Negative cancer, without lymphovascular invasion, and a negative sentinel node biopsy.

A) Describe three “adjuvant” treatments that would be potentially beneficial to her

A

Hormonal Therapy, Chemotherapy, Radiotherapy

Hormonal tx: tamoxifen, Aromotase Inhibitors

Chemotherapy:

  • Improves disease free survival in high-risk patients.
    1. Assess the risk of cancer returning, thereby causing morbidity (local recurrence) or death (ultimately the result of distant metastases because, although these can be treated, patients are never cured). www.adjuvantonline.cmo
    2. Estimate the degree to which risk can be reduced by various modalities of adjuvant treatment (hormones, chemotherapy and biological therapy), weighed up against potential toxicity.
    3. Assess the willingness of the patient to accept toxicity for the predicted benefit.
  • Agents
    • Anthracyclines - Doxorubicin and epirubicin
    • Taxanes - Paclitaxel, docetaxel and nab-paclitaxel).
    • Alkylating agents – cyclophosphamide, carboplatin
    • Antimetabolites – 5-fluorouracil, methotrexate
  • Microscopic mets.
  • Better for post-menopausal

radiotherapy

  • Radiotherapy is aimed at achieving local disease control (i.e. preventing local recurrence)
  • This case requires RT due to the surgical technique being breast conserving (wide local excision), however if she had had a mastectomy with high risk features she would also need RT
  • RT is usually external beam with either opposed tangential beams, or the more modern techniques of conformal therapy with IMRT and respiratory gating
  • Usually: 40 - 50Gy/ 15 - 25# (other fractionation schedule options and booster fields depending on the disease)
  • RT usually starts between 3 - 6 weeks after surgery
How well did you know this?
1
Not at all
2
3
4
5
Perfectly
26
Q
  1. A 50 year old woman is diagnosed with breast cancer. Her surgeon recommends a “lumpectomy” and sentinel node biopsy. The pathology report shows a 20 mm grade 3 ER positive, PR positive, Her-2 negative cancer, without lymphovascular invasion, and a negative sentinel node biopsy.
    b) Referring to the aims of each treatment and distinguishing between the aims of adjuvant systemic therapy and adjuvant local therapy.
A

Adjuvant Radiotherapy – aimed at achieving local disease control (preventing local recurrence)

Adjuvant Hormonal Therapy – Systemic control

Adjuvant chemotherapy – systemic control

How well did you know this?
1
Not at all
2
3
4
5
Perfectly
27
Q
  1. A 30 year old man presents with a 3.2mm melanoma on his right arm. A wide excision and sentinel node biopsy is performed. One node is removed from the right axilla as part of the sentinel node procedure. It contains a 0.3mm deposit of melanoma.

What are the biological argumens for and against proceeding to an axillary clearance?

A
  • Arguments for axillary clearance
    • There is a high risk of micrometastases in other lymph nodes and therefore completion lymph node dissection should be performed over partial dissection or sampling.
    • There are improved mortality rates for patients that undergo axillary dissection, compared to those who don’t.
    • Completion lymph-node dissection has improved regional disease control and allows more precise staging and prognostic information which will influence decisions regarding adjuvant systemic therapy
  • Arguments against axillary clearance
    • MSLT-II trial showed no survival benefit for patients who underwent immediate completion lymph node dissection, when compared to patients who were observed and only underwent operation when lymph node involvement was clinically detectable.
    • Side effects of axillary clearance include: wound infection, seroma formation, shoulder dysfunction, lymphoedema, paraesthesia
How well did you know this?
1
Not at all
2
3
4
5
Perfectly
28
Q
  1. A 53 year old man presents with a pearly nodule on his back that has, according to the patient “come up fairly quickly in the last 6 weeks”. It is bleeding when he rubs it with a towel.

Q1) How should you manage this problem?

A

Features of malignant melanoma

  • Increase in pigmentation
  • Increase in size
  • Bleeding
  • Ulceration or crusting
  • Spread of pigmentation
  • Satellite or in-transit lesions
  • Pain or itching

BBC – Most common but least lethal

  • Pearly appearing rolled border
  • Develop from basal cell layer skin
  • UV à Thymine dimer à damages DNA
  • Slowly growing plaque or nodule
  • skin colored, pink or pigmented
  • Spontaneous bleeding or ulceration
  • Nodular BCC: most common type of facial BCC. Shiny or pearly nodule with smooth surface. Central depression or ulceration so its edges appear rolled. Blood vessels across surface.
  • Superficial BCC: most common type in younger adults, upper trunk and shoulders, slightly scaly, irregular plaque, thin, translucent rolled border, multiple microerosion.

History

  • When did it start?
  • Where did it start? Where is it now?
  • Have you had it before?
  • How has it changed?
  • It is evolving or stable?
  • Any discharge or bleeding?
  • Pain, itch, dryness, altered sensation?
  • Any obvious triggers or exacerbating factors? (sunlight, temperature, substances, work)
  • Relieving factors?
  • What treatments have been tried?
  • Systemic symptoms? (fever, weight loss, malaise, joint pain, cough, cold, sore throat.

Examination

CCCSSSFFFTTT

Size, Shape, Site

Contours, Colour, Consistency

Temperature, Tender, Transluminate (fluid vs mass)

Fluctuate, Feel of drainage in lymph, Firm

Check all skin for other lesions

Biopsy

  • excision biopsy with 2-5 mm margins
  • punch biopsy if large lesion
  • cryotherapy (double freeze thaw technique)
  • radiotherapy (if margins where not cleared)
How well did you know this?
1
Not at all
2
3
4
5
Perfectly
29
Q
  1. A 55 year old woman presents with breast lump.

A) What clinical and mammographic features together would allow you to be sure that it is benign?

A

Benign:

Clinical features:

  • Painful
  • Fluctuating/ cyclical
  • Smooth/ regular borders
  • Firm/ rubbery
  • Mobile
  • Absence of skin changes, nipple changes and axillary lymphadenopathy
  • Discharge is milky/ clear/ green (not bloody)

Mammographic features:

  • Well circumscribed lesion
  • Large, course calcifications “clunky” (i.e. not spiculated) – absence of microcalcification
  • Fatty/ low density

Ultrasound Features:

  • Smooth margins
  • Oval
  • Wider than they are tall
  • Macro-lobulation
  • Clearly defined margins
  • Posterior enhancement
  • Anechoic lesion

note:

  • To be sure that a lesion is benign a triple assessment should be performed (clinical assessment, imaging and biopsy)
  • The imaging findings would be classified using the Breast Imaging-Reporting and Data System (BIRADS) to determine risk of malignancy and further management and investigation
How well did you know this?
1
Not at all
2
3
4
5
Perfectly
30
Q
  1. A 45 year old woman presents with a new breast lump that she has only just noticed.

A) What single investigation might allow you to reassure her, and what finding on that investigation would be most reassuring?

A

All women should undergo triple therapy at a breast clinic on the same day (Clinical Assessment, imaging and tissue biopsy) on the same day

  • 99.6% accuracy when performed by experienced personnel.

Single investigation: USS guided core biopsy

  • 97% sensitive.
How well did you know this?
1
Not at all
2
3
4
5
Perfectly
31
Q
  1. A 36 year old woman has a mastectomy and axillary clearance for a large “triple negative” breast cancer with multiple nodes involved. She is offered adjuvant chemotherapy which should provide a relative risk reduction of about 25%. Without treatment, this woman’s chances of 5 year survival are 20%.

A) what are her chances of survival with the adjuvant chemotherapy? Explain how you arrived at this answer

A

5 year survival without treatment = 20%

AR (C) = 80%

RRR = 25%

AR (T) = absolute risk in treatment group

AR (T) = AR(C) – AR(C)xRRR

= 0.8 - 0.8 x 0.25

= 0.8 – 0.2

= 0.6

Absolute risk of death at 5 years in treatment group with treatment is 60%

Thus 5 year survival is 40% in treatment group

How well did you know this?
1
Not at all
2
3
4
5
Perfectly
32
Q
  1. A 50 year old woman has a wide excision and sentinel node biopsy for a brest cancer. The pathology report shows a 30mm grade 3 ER positive, PR positive, HER-2 negative cancer, without lymphovascular invasion, and a negative sentinel node biopsy. Describe the adjuvant treatment options available to her. Refer to the different aims of adjuvant systemic therapy and adjuvant local therapy
A
  • Treatment plan of all patients should be discussed at MDT meetings

Adjuvant Radiotherapy

  • Radiotherapy is aimed at achieving local disease control (i.e. preventing local recurrence)
  • This case requires RT due to the surgical technique being breast conserving (wide local excision), however if she had had a mastectomy with high risk features she would also need RT
  • RT is usually external beam with either opposed tangential beams, or the more modern techniques of conformal therapy with IMRT and respiratory gating
  • Usually: 40 - 50Gy/ 15 - 25# (other fractionation schedule options and booster fields depending on the disease)
  • RT usually starts between 3 - 6 weeks after surgery

Adjuvant Hormonal Therapy

  • This woman’s tumour is ER +ve / PR +ve / HER2 -ve, therefore she should be given hormonal treatment to target the ER (e.g. SERM or aromatase inhibitor, depending on whether she is pre- or post-menopausal)
    • These treatments are systemic treatment
  • SERM (e.g. Tamoxifen) – used in pre- and perimenopausal women
    • Competitively binds to the oestrogen receptor and therefore limits the stimulation of growth (antagonist in the breast but agonist in the bone and endometrium
    • Main side effects: menopausal symptoms (vaginal dryness, hot flushes), increased risk of VTE and increased risk of endometrial cancer, which is significant after 5 years of treatment
    • Reduction in mortality of 26% and up to 47% reduction in local recurrence at 10 years
    • Stop treatment after 5 years and switch to aromatase inhibitor for 5 years
  • Aromatase Inhibitors (e.g. Anastrozole, Letrozole, Exemestane) – used in post-menopausal women
    • These drug block the conversion of androgens to oestrogens in peripheral adipose tissue and reduce the stimulation of the tumour
    • Main side effects: worsening menopausal symptoms (hot flushes, osteoporosis, vaginal dryness), arthralgia and myalgia
    • Patient should have their bone health check regularly – increased risk of fractures
  • Ovarian suppression (e.g. Goserelin [GnRH agonist], oophorectomy) – used in treatment of pre- and perimenopausal women
    • GnRH increases LH/FSH, which if continuous leads to sustained inhibition of gonadotropins and supressed ovarian steroidogenesis
  • Note: if she were HER2 +ve then Trastuzumab (Herceptin) would have also been included

Adjuvant Chemotherapy

  • Systemic chemotherapy should be considered due to the high grade of the lesion, even though the SNB was negative
  • Mechanism is disruption of the cell cycle
  • There are many different regimens of cytotoxic agents used to treat breast cancer
  • Examples of cytotoxic medications:
    • Anthracyclines – doxorubicin and epirubicin
    • Alkylating Agents – cyclophosphamide, carboplatin
    • Antimetabolites – 5-fluorouracil, methotrexate
    • Taxanes – paclitaxel and docetaxel
  • The use of adjuvant chemotherapy is usually reserved for those patients with poor prognostic indicators such as:
    • Large tumour size
    • Node positivity
    • Grade 3 tumours
    • Extensive lymphovascular invasionp
How well did you know this?
1
Not at all
2
3
4
5
Perfectly
33
Q
  1. How are anal fissures treated?
A

Conservative or operatively.

Conservative

  • Topical gylerceryl trinitrate (GTN) ointment 0.2-0.4%, applied 3 times a day for a month.
    • Relaxes sphincter spasm
    • Increases blood supply to fissure allowing healing
    • May cause headaches
  • Diltiazem ointment
    • Calcium channel blocker
  • Injection of botulinum toxin into the sphincter complex
  • Prevention with high-fibre diet and adequate fluids

Operative tx:

  • Lateral submucous (internal) sphincterotomy
    • Immediate relief
    • 10-15% incidence of incontinence or flatus following procedure
    • Less offered in women due to shorter sphincters and injury from childbirth
  • Anal advancement flap
    • Chronic refractory fissures
    • Avoids sphincter muscle damage of sphincterotomy
  • Lords anal stretch
    • Manual dilation of the sphincter
    • High rates of incontence due to sphincter damage – no longer offered.
How well did you know this?
1
Not at all
2
3
4
5
Perfectly
34
Q
  1. What is an anal fissure? How does it develop? What causes it to persist?
A

Anal fissure is a longitudinal tear in the mucosa and skin of the anal canal, sometimes caused in the passing of a large, constipated stool.

  • Tear is usually posterior midline of the anal margin
  • Causes acute pain and sphincter spasm, small amount of fresh blood, exacerbated by defecation
  • Loop of fear of defecation worsening the constipation
  • Development related to ischaemia
    • Rate of perfusion is inversely related to the anal pressure.
  • Primary – local trauma
    • Contripation
    • Diarrhea
    • Vaginal delivery
    • Anal sex
  • Secondary
    • Crohn disease
    • Other granulomatous diseases
    • Malignancy (anal SCC, leukemia)
How well did you know this?
1
Not at all
2
3
4
5
Perfectly
35
Q
  1. How would you go about investigating a previously well 63 yoa man who presented to you with a short history of rectal bleeding?
A
  • ACBDE
  • Haemodynamic status
    • Hypotension, tachycardia, cool peripheries, tachypnoea, decrease consciousness.
    • Resuscitation is priority
  • History
    • How much blood?
    • Duration and requency?
    • What colour?
    • Mixed with stool? Streaked on stool? Separate? Seen only on toilet paper?
    • Pain or prolapse on defecation?
    • Tenesmus?
    • Change in bowel habits?
    • Weight loss?
    • Symptoms of anaemia?
    • Past medical hx
      • Previous rectal bleeding
      • Previous colonoscopy
      • UC?
      • Bowel trauma?
      • Aortic surgery?
      • Radiotherapy for the rectum?
      • Bleeding tendency?
      • Hx of disease predisposing to upper GI bleeds (PUD, chronic liver disease)?
    • Meds
      • Anticoagulants? Antiplatelets?
      • PUD – NSAIDs/steroids/bisphosphonates
      • Infective colitis – Abx – C. Difficile
    • Fhx:
      • Colorectal cancer
      • Bowel diseases
    • Shx
      • Smoking
      • Travel hx
    • Systems review
  • Examination
    • General
      • Vitals
      • Signs of shock
      • Signs of chronic blood loss (anaemia – pallor, koilonychia, SOB
      • Signs of malignancy (cachexia, lymphadenopathy)
    • Abdominal examination
      • Tenderness/masses/organomegaly/ascites
    • Rectal examination
      • Inspect the anus
        • Anal fissue +- skin tags +- sentinel pile, haemorrhoid, fistula
      • Blood on withdrawn finger
  • Investigations
    • FBC, UEC, LFTs, COags, CRP,
    • Proctoscopy +/- sigmoidoscopy
    • Gastroscopy and colonoscopy
    • Mesenteric angiography
    • Radionuclide imaging
    • Laparotomy and subtotal colectomy (if patient unstable)
How well did you know this?
1
Not at all
2
3
4
5
Perfectly
36
Q
  1. How would you go about investigating a previously well 63 yoa man who presented to you with a short history of anaemia?
A
  • Anaemia is a state where the level of Hb in the blood is below the reference range for age and sex:
    • Men (130 – 180 g/L)
    • Women (115 – 165 g/L)
  • Iron deficiency anaemia is the most common type worldwide

Mechanisms

  1. Disorder of production
    1. Bone marrow failure - e.g. aplastic anaemia, red cell aplasia
    2. Decreased Erythropoeitin - e.g. Chronic Kidney Disease
  2. Disorder of maturation
    1. Nuclear maturation defects - e.g. B12 or folate deficiency, myelodysplasias
    2. Cytoplasmic maturation defects - e.g. iron deficiency, thalassaemia
  3. Decreased Survival
    1. Inherited defects - e.g. spherocytosis, G6PD deficiency, sickle-cell anaemia
    2. Acquired defects - e.g. Autoimmune haemolysis, malaria, DIC, TTP
  4. Sequestration in spleen – hypersplenism

History:

  • Symptoms of anaemia:
    • Fatigue, lethargy, muscle weakness, headaches
    • Dyspnoea
    • Palpitations
    • Worsening of vascular disease
    • GI disturbance – anorexia, nausea, bowel irregularity
    • Irregular menstrual pattern
  • Blood Loss and risk factors:
    • GI bleeding (PR bleeding, hematemesis, melaena, menorrhagia [if female])
    • NSAIDs, aspirin, colon polyps of previous colonoscopy, HHT
  • Gastrointestinal Surgery/ Disease:
    • Gastrectomy or ileal resection
    • Small bowel resection
    • GORD/ PUD
    • Diverticulosis
    • IBD (Crohn’s or UC) or Coeliac disease

Examination:

  • General:
    • Vitals (HR, RR, BP, Temp, SaO2)
    • Pallor
    • Jaundice, icterus
    • Angular stomatitis
    • Koilonychia
  • Abdominal
    • Hepatomegaly
    • Splenomegaly
    • Masses
    • PR exam
  • Neurological examination
  • Cardiorespiratory examination

Diet:

  • How often does he eat red meat
  • Green vegetables
  • PMHx:
    • Chronic infections or inflammatory diseases
    • Bleeding disorders
  • Meds:
    • NSAIDs, aspirin
    • Anticoagulants and antiplatelets
  • FHx:
    • Colorectal Cancer
    • GI disorders
    • Blood disorders
  • SHx:
    • Smoking
    • ETOH

Investigations:

  • FBC (Hb, MCV, WBC, Blood film, reticulocyte count)
  • Coagulation studies
    • Once anaemia is confirmed and the RBC morphology is determined, then focused investigations can be aimed at the most likely differential diagnoses
  • Iron studies (serum iron, ferritin, TIBC & transferrin), serum B12 and folate, haemolytic parameters (serum LDH, haptoglobin, bilirubin), Direct Coomb’s test, Hb Electrophoresis
  • FOBT
  • Urinalysis and Urine MCS
  • Gastroscopy and colonoscopy
  • Small bowel capsule endoscopy
How well did you know this?
1
Not at all
2
3
4
5
Perfectly
37
Q
  1. How would you go about investigating a previously well 63 yoa man who presented to you with a short history of a change in bowel habit to more frequent evacuation of loose stools with mucus?
A

History

  • Onset & duration
  • How much stool and mucus
  • Consistency and appearance of stool
  • Light coloured, floating?
  • Foul smelling
  • Bleeding?
  • Any abdominal pain or pain on defecation
  • Relation to meals?
  • Tenesmus?
  • Fever, sweats, hot and cold, shivers?
  • Unintentional weight loss? Fatigue?
  • Any recent travel?
  • Any other people unwell?
  • PMHx – Colonoscopy? Bowel surgery? IBD?
  • Laxative use? Recent abx?
  • SHx: smoking, EtOH, recreational/IV drugs, change in diet,
  • FHx: GI disease, colorectal cancer

Examination:

  • General appearance: well/unwell, nutritional status
  • Vitals (HR, RR, BP, temp, SaO2
  • Assess fluids
    • Mucous membranes
    • Capillary refill
    • JVP
  • CVS/Resp
  • Abdo exam

Investigations

  • FBC + iron studies, folate, B12 (anaemia)
  • UEC, LFT, CMP, coagulation studies
  • Stool MCS (leukocytes, occult blood, ova/parasites)
  • Stool PCR for C. Difficile toxin
  • Colonoscopy/ ?CT colonography
How well did you know this?
1
Not at all
2
3
4
5
Perfectly
38
Q
  1. What are the major risk factors for colorectal cancer?
A

Major

  • Increasing age (>60yrs)
  • Western diet (low fibre, high fat)
  • Ulcerative colitis
  • Polyposis syndromes
    • Familial adenomatous polyposis
    • Peutz-jeghers syndrome – hamartomatous polyps
  • Hereditary non-polyposis colorectal cancer (HNPCC)
  • Central obesity
  • Physical activity
  • EtOH
  • Family hx of colorectal cancer with young age at diagnosis
  • Hx of polyps =/- adenomas
How well did you know this?
1
Not at all
2
3
4
5
Perfectly
39
Q
  1. What method is used for population screening for Colorectal cancer in Australia?
A

Faecal occult blood test

  • 50-74yrs
  • For low-risk people with no symptoms
  • Screening kits are mailed to residents
  • 2 yearly – kits received 6 months within birthday at 50, 52, 54, … ,
How well did you know this?
1
Not at all
2
3
4
5
Perfectly
40
Q
  1. What is the neoadjuvant therapy for rectal cancer?
A

Chemoradiotherapy to shrink the tumour to improve the chances of surgical removal.

  • Offered to Dukes C cancers (stage III)
  • Chemoradiotherapy in rectal tumours tethered in the pelvis
  • Radiotherapy where rectal cancers extend through the bowel wall anteriorly to reduce local recurrence
  • Chemotherapy in large bowel cancer – 5-fluorouracil (5-FU) is the primary agent, given in combination with bimodulator folinic acid.
How well did you know this?
1
Not at all
2
3
4
5
Perfectly
41
Q
  1. Where are the common sites that metastatic disease from colorectal primary are found?
A
  • Liver most common
  • Lungs 2nd most common
  • Bone
  • Peritoneum
How well did you know this?
1
Not at all
2
3
4
5
Perfectly
42
Q
  1. What are the treatment options for liver metastases from a colorectal primary?
A
  • Majority of patients cannot be cured and intent of treatment is usually palliative
  • If mets are localized to one anatomical lobe it may be resected. – partial hepatectomy
    • PET scan to esclude extra-hapatic metastases
    • Exclusion criteria
      • Extrahepatic metastases
      • Radiographic evidence of involvement of the common hepatic artery, common bile duct or portal vein
      • Extensive liver involvement (>70%, or >6 segments, or involvement of all 3 hepatic veins)
      • Inadequate predicted post-resection functional hepatic reserve
      • Patient not fit for surgery
  • Oral dexamethasone may temporarily reduce metastatic tissue edema and relieve symptoms
  • Neoadjuvant chemotherapy to allow a tumour to become resectable
  • Radiofrequency ablation
  • Hepatic intra-arterial chemotherapy
  • Radiotherapy
How well did you know this?
1
Not at all
2
3
4
5
Perfectly
43
Q
  1. What is the adenoma-carcinoma sequence?
A
  • Adenoma-carcinoma sequence refers to a stepwise pattern of mutation in a cell or group of cells that results in cancer
  • Mutations include
    • Activation of oncogenes from proto-oncogenes (e.g. K-RAS)
    • Inactivation of tumour suppressor genes (e.g. p53, APC)
  • The classic adenoma-carcinoma sequence accounts for 80% of sporadic colon tumours and typically includes mutation of APC early in the process
  • Supporting evidence of this model [Uptodate]
    • Early carcinomas are frequently seen within large adenomatous polyps, and areas of adenomatous change can often be found surrounding human CRCs
    • Adenomas and carcinomas are found in similar distributions throughout the large bowel, and adenomas are typically observed 10 to 15 years prior to the onset of cancer in both sporadic and familial cases.
    • In animal models, adenomas develop before carcinomas, and carcinomas develop exclusively in adenomatous tissue
    • The ability to reduce the incidence of CRC through the removal of polyp has been shown in controlled trials in humans.
How well did you know this?
1
Not at all
2
3
4
5
Perfectly
44
Q
  1. What is Crohn’s disease?
A

chronic relapsing inflammatory disorder of any part of the gastrointestinal tract (though nearly always small or large bowel

  • 60% under 25 years of diagnosis
  • Skip lesions
  • Small bowel 50%, large bowel 20%, both 30%
  • Terminal ileum most common
  • Commonly affects the perianal region regardless of large bowel involvement
  • Transmural inflammation
    • May partially obstruct, fistulate or perforate, whereas this rarely occurs in UC.
  • With each exacerbation, previously affected or new areas may become involved.
  • Pathophysiology
    • Transmural inflammation
    • Marked thickened inflammatory oedema, especially in the submucosa.
    • Fissured ulcers à cobblestoning
    • Granulomas
      • Multinulceate giant cells
    • Fibrosis à elongated strictures
  • Requires histological and microbacterial differentiation from yersinia ileitis and tuberculosis
  • Serosal inflammation à adhesions, perforation, fistulae
  • Perianal inflammation 15%
    • Recurrent perineal abscess, characteristic bluish boggy ‘piles’.
  • Ix
    • Colonoscopy & biopsy
    • Barium follow through
  • Tx
    • Anti-inflammatories
      • 5-ASA compounds – sulfasalazine, mesalazine
      • Corticosteroids
    • Immunomodulators
      • Azathioprine and 6 -mercaptopurine
      • Methotrexate
      • Infliximab
    • Other supportive therapy
      • Antidiarrhoeal and antispasmodics
      • Dietary modification
        • Liquid/low fibre in those with obstructive symptoms
        • Supplementary calories
        • Iron and vitamins
  • Surgery
    • Not curative in crohns as operating on one portion of the bowel does not prevent recurrences elsewhere
  • Crohns fissure – atypical
    • Heaped up edges
    • Multiplicity
    • >1cm
    • Not 6oClock
How well did you know this?
1
Not at all
2
3
4
5
Perfectly
45
Q
  1. What is diverticular disease?
A
  • Colonic circular muscle is thickened because of an excess of elastic tissue between muscle fibres rather than muscle hypertrophy
  • Hypersegmentation -> causes peristatltic waves at each other -> high intraluminal pressure in short segments -> mucosa herniates through weak points
How well did you know this?
1
Not at all
2
3
4
5
Perfectly
46
Q
  1. What is diverticulitis? How does it present clinically?
A
  • Chronic grumbling diverticular pain
    • Periverticular inflammation is chronic, low-grade, and recurrent. Local irritation provokes bowel wall spasm, causing bowel pain and erratic bowel habit.
    • Mild iliac fossa tenderness and faecal loading
    • Endoscopy or imaging
    • Relieved by taking a high-fibre diet
  • Acute diverticulitis
    • Continuous left iliac fossa pain, pyrexia, tachycardia.
    • Left iliac fossa tenderness to obvious local peritonitis
    • Abx: ciprofloxacin and metronidazole
    • Bowel rests with NBM
  • Hinchey classification of abscess and perforation
    • Stage 1: small pericolic or mesenteric abscess <4cm
    • Stage 2: large, pelvix abscess
    • Stage 3: small perforation causing gaseous and purulent peritonitis
    • Stage 4: free rupture with faecal peritonitis
  • Pericolic abscess
    • Fails to reverse with abx
    • Persistent pain, swinging pyrexia, incomplete obstruction due to spasm of bowel wall muscle.
    • May drain into bowel causing diarrhea
    • CT scan
    • Abx
  • Diverticular perforation
    • Acute abdomen
    • Sub-diaphragmatic gas on CT or Xray
    • Conservative treatment – percutaneous drain, bowel rest for Hinchey 3 but not Hinchey 4
    • Hinchey 4 perforation requires parenteral antibiotics and laparotomy for peritoneal toilet, diversion of faecal stream and resection of diseased bowel.
  • Fistula formation into other abdominal or pelvic structures
    • Occurs when an inflamed diverticulum lies close to another hollow viscus
    • Large bowel and small bowel causes diarrhea
    • Vesico-colic fistula fistula causes pneumaturia and severe UTI
  • Intestinal obstruction
    • Bowel obstruction due to acute inflammatory thickening, muscle hypertrophy and spasm.
    • Incomplete obstruction is more common and presents as severe constipation.
    • Chronic diverticular inflammation sometimes causes local fibrous strictures, particularly in the sigmoid à which cause intermittent bouts of constipation when the stool is dry.
  • Acute rectal hemorrhage
    • Not common.
    • Typically complains of having passed a mass of fairly fresh blood instead of the expected stool and is admitted to hospital urgently.
    • Ddx: ischaemic colitis
How well did you know this?
1
Not at all
2
3
4
5
Perfectly
47
Q
  1. What are the major complications that can result from diverticulitis?
A
  • Spreading pericolic inflammation
  • Pericolic abscess
  • Intraperitoneal perforation
  • Fistula formation into other abdominal or pelvic viscera
  • Bowel-to-bowel adhesions
  • Fibrous strictures of the bowel
  • Acute haemorrhage (which tends to occur without inflammation)
How well did you know this?
1
Not at all
2
3
4
5
Perfectly
48
Q
  1. What are haemorrhoids and how are they classified or described?
A

Definition:

  • Engorged normal vascular structures located in the submucosal layer of the anal canal
  • Arise from a plexus or cushion of dilated arteriovenous channels ad connective tissue that drain into the superior, middle or inferior hemorrhoidal veins
  • They are normally located in the left lateral, posterior and right anterior positions (3,7,11 oClock).
  • They only become an issue if they are symptomatic

Classification:

  • First degree (or grade 1): never prolapse
  • Second degree (or grade 2): prolapse during defecation and then return spontaneously
  • Third degree (or grade 3): remain outside the margin unless replaced digitally.

Anatomical classification :

  • Most considered ‘internal’ because they are covered by glandular mucosa.
  • Large neglected haemorrhoids may extend beneath stratified squamous epithelium so their lower part becomes covered by skin. ‘intero-external’ haemorrhoids.
How well did you know this?
1
Not at all
2
3
4
5
Perfectly
49
Q
  1. How are symptomatic haemorrhoids treated?
A

Conservative management:

  • Prevetion
    • High fibre diet
    • Avoid straining
    • Spend minimal time defecating
  • 3rd degree
    • Patient replacing the prolaping haemorrhoids digitally after defecation
  • Creams, suppositories, and other topial preparations available
    • Some contain local anaesthetic agents or steroids

Surgical management

  • Injection of sclerosants or banding
    • First degree and most second degree as outpatient
  • Sclerotherapy
    • 1-3ml of mildly irritant solution – 5% phenol in oil injected submucosally around the pedicles of three major hemorrhoids in the insensitive upper anal canal à fibrotic reaction à obliterates haemorrhoidal vessels and causing atrophy of the haemorrhoids.
    • Repeated 4-6wks 2-3 times.
  • Banding
    • Cone of mucosa kist above the haemorrhoid neck is drawn into a banding instrument, often by suction, and tight elastic bands released around the base of a cone, constricting the haemorrhoidal vessels.
    • Haemorrhoid gradually shrinks.
  • Haemorrhoidectomy
    • Haemorrhoidal excision
      • Third degree or tx failed
    • Miligan and morgan
      • Haemorrhoidal masses and overlying mucosa = skin excised
      • Healing by secondary intention and wound contraction
    • Stapeled hemorrhoidectomy
      • Popularity for large haemorrhoids
      • Excising ring of low rectal mucosa. Including the engorged necks of the piles
      • The metal staple line remains in situ.
  • Hemorrhoidal artery ligation operation
    • New procedure
    • US each artery supplying haemorrhoid, encircle with stitch via the insensitive lower rectal mucosa to cut off its blood supply -à haemorrhoids shrink, bleeding and local symptoms ablate, although skin tags remain.
How well did you know this?
1
Not at all
2
3
4
5
Perfectly
50
Q
  1. What are the indications for elective surgery in a patient with Ulcerative Colitis?
A

Surgery needed for 20% of patients with UC

  • Urgent tx of fulminant cases which fail to respond to intensive medical treatment.
    • ‘failure to respond’ has no precise definition but there should normally be improvement after a week of intensive therapy
    • Acute cases which progress to toxic megacolon, perforation or major hemorrhage
    • Patients with chronic disabling symptoms of intractable diarrhea with urgency, recurring anemia and failure to maintain adequate weight
    • Children with failure to thrive and retardation of growth (both are exacerbated by corticosteroids)
    • Patients with longstanding colitis who develop dysplasia or malignancy
51
Q
  1. What are indications for elective surgery in a patient with Crohn’s?
A
  • Acute complications e.g. abscess, perforation
  • Persistent local ileal disease
  • Intolerable long-term obstructive and other symptoms. E.g. abdominal pain, perianal disease, general ill-health
  • Enterocutaneous fistulae and symptomatic internal fistulae
  • Failure to respond to medical treatment

Note: surgery is not considered curative in Crohns

52
Q
  1. What are the indications for emergency surgery in a patient with inflammatory bowel disease?
A
  • Major Stenotic disease
  • Major Hemorrhage
  • Perforation, peritonitis
  • Toxic megacolon
  • Intra-abdominal abscess with sepsis
  • Intestinal obstruction (complete)
53
Q
  1. You are the ED intern on duty. You are asked to see a 20yoa man who has presented with perianal pain that started 24 hours ago and has got worse despite appropriate analgesia. the pain is worse when he walks or sits. This has not happened before. He has no other relevant history. on examination you find a fluctuant, warm, red 3x3cm swelling at the anal margin. What is the likely diagnosis?
A

Most likely ddx: perianal abscess

  • Other ddx
    • Pilonidal abscess
    • Anal fissure
    • Thrombosed haemorrhoids
    • Infected epidermoid cyst
    • Perianal hidranitis suppurativa

Pathophysiology:

  • Begin as acute purulent infections of anal glands. These lie in the intersphincteric space between external and internal crypts near the dentate line.
  • Infection spreads in 3 directions
    • Downwards towards anal verge -> perianal abscess (80%)
    • Outwards through external sphincter -> ischiorectal abscess (15%)
    • Upwards -> supralevator abscess involving pararectal tissues and pelvic floor

Tx:

  • Very early -> oral antibiotics
  • Surgical drainage
  • Antibiotics only indicated when there is spreading infection
  • Swab MCS
    • Staphylococcus
    • E. Coli -> suggests underlying fistula

If large packing or placement of drain to keep neck of cavity open while granulation tissue fills the space from its depths

54
Q
  1. You are the ED intern on duty. You are asked to see a 20 yoa man who has presented with perianal pain that started 24 hours ago and has got worse despite appropriate analgesia. the pain is worse when he walks or sits. This has not happened before. He has no other relevant history. On examination you find a fluctuant, warm, red 3cmx3cm swelling at the anal margin. What aeitological factors are associated with this condition?
A

Likely to be perianal abscess

  • Originates from infected anal crypt gland
  • 8-10 anal crypt glands arranged circumferentially within the anal canal at the dentate line
  • Glands penetrate the internal sphincter and end in the intersphincteric plane
  • Abscess develops when the narrow anal gland crypt becomes obstructed with inspissated debris à bacterial growth à abscess formation
55
Q
  1. What is Ulcerative Colitis?
A
  • Inflammatory disorder of the mucosa and submucosa of the large bowel only.
  • Characterised by acute exacerbations and intervening periods of quiescence or chronic low-grade activity.
  • Symptom level correlates with disease level.
  • Extracolonic features
    • Anaemia
    • Arthropathy
    • Inflammation of eyes, skin, and biliary tract.
  • Pathophysiology
    • Colonic mucosa becomes acutely inflamed .
    • Neutrophils accommodate the lamina propria and within the tubular colonic glands à crypt abscesses
    • Sloughing of the mucosa à small superficial ulcers
    • Ulcers coalesce into extensive areas of irregular ulceration;
    • Residual islands of intact but oedematous mucosa project into the bowel lumen; pseudopolyps
    • Quiescent periods
      • Acute inflammation resolves and the mucosa regenerates
      • Lamina propria remains swollen by chronic inflammatory infiltrate of lymphocytes and plasma cells
      • Goblet cell depletion
    • Dysplasic changes à adenocarcinoma
    • Malignancy risk 5% after 10 years of colitis
    • Longstanding colitis à fibrosis à smoothing out haustra and shortened colon
  • Clinical features
    • Loose blood-stained stools streaked with mucus.
    • Abdominal cramping prior to loose stools
    • Weight lodd, anaemia, arthropathy, uveitis, iritis, skin lesions, sclerosing cholangitis
    • Fulminant colitis
      • Dehydration, severe electrolyte disturbance and blood loss
      • Urgent hospitalization
      • Resuscitate fluid, electrolyte and blood replacement.
      • Sigmoidoscopy and biopsy to diagnose
      • Stool for MCS
      • Abdo Xray to monitor for dilatation
    • Toxic megacolon – colon dilates massively and patchy necrosis occurs
      • High fever, marked tachycardia and dehydration
  • Ix
    • Inc. ESR, CRP, dec Hb
    • Contrast radiology
      • Barium enema to assess the extent and degree of colonic involvement
    • Endoscopy
      • Unprepared flexible sigmoidoscopy
      • Direct inspection and sample biopsy
  • Mx
    • Local corticosteroid dor 5-ASA preparations (suppositories, foam or liquid enema
      • Left-sided active disease
    • Systemic corticosteroids
      • Suppress moderate or severe exacerbations
    • Oral aminosalicylate preparations e.g. sulfasalazine, mesalazine or olsalazine
      • Long term maintenance therapy to minimize relapse
    • Surgical removal of the colon
      • Emergency operation: incipient or actual perforation, serious haemorrhage, failure of fulminant colitis to improve medial tx
      • Elective operation: failure of medical treatment, risk of malignancy
56
Q
  1. This lesion is found at colonoscopy in the proximal colon. What is it? How should it be treated? Is follow up or surveillance needed?
A

Polyp = morphological decription of any localized lesion protruding from the bowel mucosa into the lumen.

Note: it does not specify any pathology

  • The term polyp is simply a morphological description for any localised lesion protruding from the bowel mucosa into the lumen (it does not imply any specific pathology)
  • The most common and most significant type of polyp are adenomatous
  • Adenomas all have the potential for malignant change, however in general it takes 5-10 years to progress to invasive cancer
  • As a general rule the larger the polyp the more likely it is to be malignant
  • Polyps can arise in any part of the larger bowel but ¾ of polyps arise in the rectum and sigmoid

Classification of colonic adenomas:

  • Tubular adenomas (pedunculated or sessile)
    • Retain the normal tubular form of the colonic mucosa
    • Have the least potential for malignant transformation
  • Villous adenomas (usually sessile and papilliferous [frond-like])
    • Tend to secrete mucus
    • More dysplastic than tubular adenoma, therefore greater malignant potential
    • Malignant potential is proportional to size
  • Tubulo-villous adenomas (most are pedunculated)
    • Majority of colonic polyps
    • Intermediate between tubular and villous adenomas

Symptoms and Signs:

  • Most are asymptomatic
  • Rectal bleeding or Iron deficiency anaemia
  • Mucus production (esp. villous adenomas)
  • Tenesmus (painful urge to defecate)
  • Prolapse
  • Hypokalaemia (rarely)
  • Obstruction or intussusception (rarely)

Treatment

  • Polypectomy (endoscopic snare)
57
Q
  1. This 22yoa female patient presents with vague intermittent right sided abdominal discomfort and some mild diarrhoea. What abnormality is evident here in the terminal ileum and right colon and colonoscopy? What histopathological finsgins are anticipated in the biopsy findings of this lesion? What are possible diagnosis?
A

Features suggestive of crohns disease:

  • Specific
    • Aphthous ulcers (superficial ulcerations)
    • Cobblestoning
    • Discontinuous lesions – skip lesions
  • Supportive but not specific
    • Normal rectum
    • Presence of normal vasculature adjacent to affected tissue
    • Isolated involvement of the terminal ileum

Histopathology:

  • Transmural inflammatory infiltrate
  • Lymphoid aggregates
  • Granulomas with typical giant cells
  • Fissured ulcers
58
Q
  1. This lesion is found at colonscopy in the sigmoid colonn of an 85 yoa male admitted acutely with a near complete large bowel obstruction. what is the lesion? what initial treatment is most appropriate for this patient whose CT scan shows a large number of bilobar hepatic matases? what are the major risks of this approach?
A
  • Colorectal cancer

Initial treatment of advanced colorectal cancer:

  • Supportive care à analgesia, anti-emetics, possible decompression with NG
  • Pt age and bilobar hepatic mets à unlikely candidate for curative procedure by liver or bowel resection
  • Colonic stenting allows for decompression
    • Risks
      • Perforation
      • Migration
      • Re-obstruction
  • Diverting colostomy +/- palliative resection
    • Risks of colostomy
      • Peristomal skin-breakdown
      • Parastomal hernia,
      • stomal stenosis,
      • bleeding,
      • prolapse
    • risks of resection
      • haemorrhage
      • surgical site infection
      • anastomotic leak à faeculent peritonitis
59
Q
  1. What is the obvious abnormality on this X-ray and in this operative photo? In the presence of symptoms of large bowel obstruction what initial treatment options exist? What can be done to prevent recurrence?
A
  • Sigmoid volvulus
    • Sigmoid rotates around narrow mesentery, causing obstruction
  • Initial tx
    • Sigmoidoscopy (most cases)
      • If sigmoid volvulus diagnosed, sigmoidoscope is gently passed as far as possible into the rectum and flatus tube inserted through it. The end of the tube is gently manipulated through the twisted bowel into the obstructed loop. If successful there is a gush of liquid faeces and flatus, relieving the obstruction
      • Flatus tube may be left in situ for 24hrs to maintain decompression, discourage re-twisting and allow recovery of the vascular supply of the bowel wall.
    • Gastrogaffin enema examination
      • If volvulus unconfirmed by X-ray and sigmoidoscopy
      • Pressure from the enema may cause the bowel to untwist, releasing flatus and faeces,
    • Surgery
      • Most caes, the bowel is still viable but sigmoid colectomy usually required to prevent recurrence
      • Double-barrelled colostomy rather than primary anastomosis in dilated and unprepared colon
      • Sigmoid colectomy or suture the bowel to the abdominal wall may prevent recurrence.
  • Prevention of recurrence
    • Surgical interventions mentioned above
    • Adequate fiber, fluids, exercise, stool softeners,
    • Prevention of chronic constipation
60
Q
  1. This is a photo of a patient with a loop defunctioning colostomy. What is the abnormality seen here? What is the mechanism? What treatment options are there?
A

Parastomal hernia

  • Most frequent complication following colostomy or ileostomy (up to 50% of pts)
  • Incisional hernia
    • Protrusion of abdominal contents through abdomen wall defect (trephine) created curing ostomy formation
    • End-colostomy has highest rate of hernia formation
  • Pt specific risk factors:
    • Advanced age
    • Wound infection
    • Chronic or reccurent increases in intra-abdominal pressure, chronic obstructive pulmonary disease
    • Obesity
    • Weight gain after ostomy construction
    • Malnutrition
    • Glucocorticoids,
    • Immunosuppression
    • Malignancy
    • Inflammatory bowel disease.
  • Technical factors
    • Emergency stoma placement
    • Surgical technique for ostomy construction (open, laparoscopic)
    • Abdominalwall strength.
    • Diameter of trephine in abdominal wall fascia

Treatment options

Ideal: re-join bowel and remove stoma but not always possible.

Supportive care – for pts with no or mild symptoms.

  • Education about signs and symptoms of bowel obstruction, bowel strangulation/infarction
  • Stoma care – stoma belt to hold stable position and lessen shearing forces.

Hernia repair

  • Indications: acute parastomal hernia complications or chronic symptoms affecting QOL.
    • Stoma dysfunction and leakage not responsive to conservative measures
    • Skin breakdown
    • Recurrent partial bowel obstruction
  • Mesh
    • Reduces recurrent hernias (subcutaneous or extraperioneal)
  • Surgical approach
    • Primary repair (high recurrence)
    • Mesh repair (most common)
    • Relocation of the stoma
61
Q
  1. what is the lesion visible here at colonoscopy and what has been done? what is the possible histology? What additional endoscopic maneouvre is needed before completion of the colonosopy?
A

Snare excision polypectomy

Possible Histology:

  • Adenoma/ adenocarcinoma
    • Tubular
    • Villous
    • Tubulo-villous
  • Hyperplastic
    • Metaplastic mucosal polyp
    • Lymphoid aggregation
  • Hamartomatous
  • Inflammatory
    • Pseudopolyp (of severe UC)

Additional endoscopic manoeuvre:

  • Retrieval of the polyp (via aspiration or net retrieval)
  • Retroflexion of the colonoscope (to visualise the right colon and also distal rectum)
62
Q
  1. This patient presents with anal discharge and pain. What is the likley nature of this lesion? What does the initial management involve? What is the role of surgery?
A

Likely diagnosis: anorectal abscess - Perianal Abscess

Nature of the lesion

  • Anorectal abscess is an acute purulent infections of the anal glands, due to obstruction of the anal crypts – the infection spreads and most often becomes a perianal abscess, which discharges to the skin around the anus.
  • Anal fistula usually develop as a complication of anorectal abscesses. The fistula is an abnormal connection between 2 epithelial surfaces and consists of a chronically infected tract which passes from the internal opening at the level of the dentate line to the external opening on the perianal skin

Presentation

  • Severe, constant anorectal pain
  • Constitutional symptoms: fever + malaise
  • Purulent rectal drainage if draining spontaneously
  • Erythematous, fluctuant or indurated skin
    • Deeper abscesses may be only felt on PR or observed on imaging

Initial management

  • Analgesia
  • Examination under anaesthetic (EUA)
  • Incision and drainage of abscess
  • Abx may be used as adjunct to surgical drainage in severe cases (gentamicin + amoxiciilin + metronidazole)

Role of surgery

  • Some larger abscesses may require general anaesthetic for incision and drainage, including ishiorectal abscesses
  • Surgery is needed to cure a fistula
    • Goal: remove tract while preserving continence
    • Superficial fistula: mx with fistulotomy (opening the length of the tract to the skin surface to allow the would to heal slowly
    • More severe cases: a seton loop (thread) through the tract and loosely tied allows free drainage of pus and after a long period of quiescence the Seton in removed in the hope the tract will close
    • Worst case – perineal resection and permanent end colostomy
63
Q
  1. This patient presented with severe anal pain after lifting a heavy piece of furniture. He feels this perianal swelling but does not present to his GP for a couple of days. What is the diagnosis? What is he treatment?
A

Most likely: thrombosed external haemorrhoids or strangulated internal haemorrhoids (due to prolapse)

  • External haemorrhoids are painful because they arise below the dentate line and are therefor covered in somatically innervevated anoderm
    • External haemorrhoids are vascular cushions comprising the inderior haemorrhoidal veins
    • When venous pressure increases and the haemorrhoids become engorged, thrombosis may develop
  • Prolapsed internal haemorrhoids become painful due to strangulation.
    • Grading
      • Grade I – no prolapse
      • Grade II – prolapse but haemorrhoids return spontaneously
      • Grade III – prolapse but haemorrhoids have to be manually reduced.
      • Grade IV – prolapsed and irreducible
    • Most likely grade III or IV
  • Treatment
    • Thrombosed external haemorrhoid
      • If untreated the pain will persist for 1-2 weeks, worse on defecation
      • Most cases subside over a few days and only require oral analgesia
      • If pain prolonged or severe: the thrombosis may be incised and drained under local anaesthesia (some surgeons prefer this as first line therapy)
    • Strangulated internal haemorrhoid
      • Symptomatic relief with bed rest, ice packs and topical anaesthetic
      • Some surgeons favour urgent haemorrhoidectomy, accepting the slightly higher risk of complication in the exchange for a more rapid recovery and a shorter hospital stay.
      • Prophylactic antibiotics should be given to cover the operation due to risk of infection in any necrotic tissue
64
Q
  1. This man had an abscess drained in the perianal region.

A) What is the common presentation of this problem

B) What is the first line of management?

C) What bacteria are usually cultured?

D) What is the usual cause of this?

A
  1. Q1: What is the common presentation of this problem?

Presentation:

  • Severe constant perianal pain
  • Low grade fever and tachycardia
  • Purulent rectal discharge (If draining spontaneously)
  • Erythematous, fluctuant mass with indurated skin.
  1. Q2: What is the first line of management?

Initial management

  • Analgesia
  • Examination under anaesthetic (EUA)
  • Incision and drainage of abscess
  • Abx may be used as adjunct to surgical drainage in severe cases (gentamicin + amoxiciilin + metronidazole)
  1. Q3: What bacteria are usually cultured?

Aerobic bacteria:

  • Staphylococcus aureus (skin cause)
  • Streptococcus pyogenes (skin cause)
  • E. Coli (GIT)

Anaerobic

  • Bacteriodes fragilis
    1. Q4: What is the usual cause of this?

Acute purulent infections of the anal glands (cryptoglandular infections) that lie in the intersphincteric space (the plane between the internal and external anal sphincters), which drain into tiny pits near the dentate line (anal crypts). These narrow ducts can easily become obstructed when the crypts are occluded by food matter, oedema from trauma due to hard stools or foreign body, or as a result of adjacent inflammatory processes (e.g. Crohn’s disease – commonly causes perirectal abscesses)

65
Q
  1. This patient had an endoscopic mucosal resection (EMR) of a colonic polyp.

A) What are the complications of this procedure

B) What is the likely pathology here?

C) What would normally occur if there is found to be a tiny focus of completely excised, well differentiated invasive cancer in the specimen that is far from the resection line?

A

EMR: removes larger and flatter polyps in the large bowel.

  • Carbon dioxide or air is pumped into the bowel to allow good view.
  • Blue dye may be sprayed onto the polyp to help outline the edges of the polyp to assist in complete removal of the polyp.
  • Small amount of gelofusion injected under the polyp to lift it away from the deeper muscle layers of the bowel wall. This increases the chance of a ccomplete revmoval of the polyp and reduces the risk of complications
  • Wire loop then passed down the colonoscope and over the polyp. Can then be removed.
  • Diathermy to seal the area.

1. What are the complications of this procedure?

Bleeding (2-11%)

  • Can be managed endoscopically w injection of adrenaline or haemoclips

Perforation

  • Rare, surgical intervention

Failure to resect polyp entirely.

Pain and discomfort

Reaction to the drugs

2. What is the likely Pathology here?

EMR used for sessile polyps. Most commonly villous adenoma.

3. What would normally occur if there is found to be a tiny focus of completely excised, well differentiated invasive cancer in the specimen that is far from the resection line?

  • Definitive treatment of polyps may be performed via endoscopy if
    • Clear margin of excision (1-2mm)
    • Moderate – well differentiated cancer
    • Absence of lymphovascular cancer
    • Complete removal assessed endoscopically
  • Unfavourable features -> age, health, pt wishes and anatomical location polyp
  • Surveillance colonoscopy at 12 months as per Australian guidelines.
66
Q
  1. This is a photo of the anal canal takn at colonoscopy by retroflexion of the scope.

A) What does this show?
B) How do these present clinically?

C) How are they treated?

A
  1. Q1. What does this show?

Internal haemorrhoids.

  1. Q2. How do these present clinically?

Cardinal features:

  • Bleeding, anal pruritus, prolapse, pain (thrombosis and strangulation)
    • Mild faecal incontinence, mucus discharge, wetness, sensation of fullness in the peranal area.
    • Itching due to
      • Mucus deposition on perianal skin (from columnar epithelium covering internal haemorrhoids)
      • Leakage of rectal contents
      • Skin tags associated with external haemorrhoids à prolonged contact of faecal material with perianal skin à local irritiation
  • Grades
    • I: visualized of anoscopy, bulge into lumen, no prolapse below dentate line
    • II: prolapse out of anal canal with defecation/straining, reduce spontaneously
    • III: prolapse out of anal canal with straining, requiring manual reduction
    • IV: irreducible, may strangulate
  1. Q3. How are they treated?

Conservative:

  • Increase dietary fibre and avoid straining. Do not ignore urge to opem bowels.
  • Topical analgesia + corticosteroids (hydrocortisone + lidocaine)

Interventional management

  • Sclerotherapy
    • Proctoscope inserted into rectum
    • Irritant solution of 5% pheno in oil injected submucosally around the pedicles of three major haemorrhoids
    • Fibrotic reaction à obliterating the haemorrhoidal vessel and causing atrophy of the haemorrhoids à atrophy of haemorrhoids
    • Repeated 2-3 times at 6-8wks
  • Banding
    • A cone of mucosa just above the haemorrhoidal neck is drawn into a banding instrument, often by suction
    • Tight elastic bands released around the base of the cone, constricting haemorrhoidal vessels à ischemic necrosis
    • Note: not placed on the stalk of the haemorrhoids due to the somatic innervation of anal skin
  • Haemorrhoidal artery ligation operation (HALO)
    • Use US to locate each artery supplying the each haemorrhoid and encircle it with a stitch via the insensitive lower rectal mucosa to cut off its blood supply.
    • Haemorrhoids shrink, bleeding and local symptoms ablate, skin tags remain.

Surgical management

  • Haemorrhoidectomy
    • Milligan and morgan
      • Haemorrhoidal mass excised with overlying mucosa and skin.
      • Skin and mucosa heal by secondary intention
    • Stapled haemorrhoidectomy
      • Large haemorrhoids & mucosal prolapse
      • Excising a ring of low rectal mucosa including the engorged necks of piles.
      • The metal staple line stays in situ
67
Q
  1. This patient presents with perianal itching.

A) What are the lesions you can see in the perianal area?

B) What is the responsible agent that causes this condition?

C) What treatments are available?

A
  1. Q1. What are the lesions you can see in the perianal area?

Most likely = perianal warts (condylomata accuminata)

Pain, bleeding, itch, embarrassment

  1. Q2. What is the responsible agent that causes this condition?
  • HPV types 6 and 11
  • Sexually transmitted, vertical. More likely from visible warts
  • Increased risk of anal canal carcinoma
  1. Q3. What treatments are available?

Topical

  • Podophyllotoxin solution or cream
  • Imiquimod cream
  • Sinecatechins ointment

In clinic

  • Cryotherapy
  • Podophyllin resin
  • Trichloroacetic acid applications
  • Electrosurgery
  • Curettage and scissor or scalpel excision
  • Laser ablation
  • 5% fluorouracil cream
68
Q
  1. How do you go about assessing a 40 year old woman who presents with a lump in her lateral neck?
A

History

  • Neck mass in adult > 40 to be considered malignant until proven otherwise
  • Mass growth pattern:
    • Rapid expansion à infectious, lymphoma
    • Little change à Benign neoplasm
    • Fluctuant with time à congenital cysts
  • Pain à Neural invasion, rapid growth
  • Voice change, hoarseness, dysphagia, otalgia à cervical lymph node mets from underlying aerodigestive tract malignancy
  • Constitutional symptoms: Fever, night sweats, weight loss
    • High spiking fever à infection
  • Social hx: Tobacco, EtOH, illicit drugs (IVDU), HIV status, occupational hx, recent travel

General examination

  • Oral cavity and oropharynx
  • Nasopharyngeal examination with mirror/ fiberoptic endoscope
  • Examination of skin on head and neck à primary skin malignancy
  • Palpate thyroid gland
  • Abdominal examination: Spleen, liver, masses

Anatomical considerations:

Midline structures:

  • Hyoid
  • Thyroid/ cricoid cartilages
  • Thyroid isthmus
  • Trachea

Mass localisation

  • Pre-auricular and angle of jaw: Salivary/ lymphoid tissue in parotid system
  • Central: Thyroid or malignant tissue, dermoid cyst
  • Antiot aspect of SCM: Lymphoid tissue, second branchial cleft cyst
  • Posterior triangle: Lymphoid tissue, likely malignant
  • Supraclavicular: Malignancy from lung/ gynae/ GI source

Characteristics:

  • Reactive: Discrete, mobile, firm/ rubbery, slightly tender
  • Malignant: Rock hard, fixed, matted
  • Infected: Isolated, asymmetric, tender, warm, erythematous, possibly fluctuant
  • Congenital cystic: Soft, ballotable, mobile
  • Carotid body tumour/ vagal schwannoma: Moves side to side, not up and down, firm, lateral mass
  • Vascular lesion: Pulsatile, bruit present
  • Thyroid: Immobile midline, elevates with swallowing à thyroglossal duct cyst, thyroid tumour

Investigations:

  • Blood:
    • FBC
    • CRP
    • EBV/CMV/ HIV serology – as guided by history
    • Other serological test as guided by history
    • ? Blood cultures if febrile
  • Imaging:
    • USS
    • Contrast enhanced CT
    • MRI
  • Histology:
    • FNA
    • Core biopsy
    • Incisional/ excisional biopsy
69
Q
  1. What are the main types of thyroid cancer and what is their prognosis?
A
70
Q
  1. What are the main treatment options for a patient who presents with a toxic goitre (hyperthyroidism associated with an enlarged over active thyroid)?
A

Symptom control

  • Propranolol
    • Controls distressing symptoms of thyrotoxicosis
    • Can be used until antithyroid drugs take effect

Antithyroid drugs

  • Carbimazole
    • Block hormone synthesis
      • prevent TPO from coupling and iodinating the tyrosine residues on thyroglobulin, and therefore leads to reduced thyroid hormone production
    • Risk of rash and agranulocytosis
      • Monitor WCC
  • Propylthiouracil
    • Poor control, frequent relapses, other side effects
  • Block and replace regimen
    • Pts given higher dose of carbimazole to completely block thyroid hormone production, together with a standard replacement dose of thyroxine

Radioiodine therapy

  • I131 or I125 given oral capsules
  • Middle-aged or elderly pts
  • Contraindicated in pregnancy
  • Iodine taken up by thyroid gland, beta-particle emission destroys the active tissue over weeks/months
  • Anti-thyroid drugs usually continued until radioiodine achieves effect
  • Isotope excreted by kidneys, faeces, sweat, saliva, breath.
  • Risk of exposing others to radiation during treatment
  • Tends to make thyrotoxic opthalmopathy worse

Thyroidectomy

  • Indications
    • When a quick and effective cure is desired that avoids long-term drug therapy. i.e. Graves in young pts.
    • When anti-thyroid drugs have proven unsatisfactory and radioiodine is unsuitable
    • In toxic multinodular goiter.
    • Toxic solitary nodules (‘hot nodules’)
  • Laryngoscopy to assess nerve function – risk of recurrent laryngeal nerve injury
  • Requires thyroxine replacement due to hypothyroidism
  • Subtotal thyroidectomy
    • Remove enough tissue to render the patient euthyroid whilst preserving enough to prevent hypothyroidism
71
Q
  1. This patient has become aware of a skin abnormality for years that has developed hair growth during adolescence
    1) what is the abnormality
    2) what is it’s significance
    3) what treatment is needed?
A
72
Q
  1. This is the abominal photograph of an 80 year old woman who presents with a painful lump in the right groin. She has had it for 24hours.
    1) What could it be? She has had generalised colicky abdominal pain from about 12 hours and some vomiting/
    2) What is the most likely diagnosis here?
    3) what is the most appropriate treatment strategy?
A

Most likely to be an inguinal hernia or femoral hernia that has become obstructed or strangulated

Differential Diagnosis of a Groin Lump:

  • Inguinal Hernia
  • Femoral Hernia
  • Inguinal lymphadenopathy
  • Saphena Varix
  • Hydrocele of the canal of Nuck
  • Psoas Abscess/ Psoas bursitis
  • Femoral aneurysm/ pseudoaneurysm
  • Neuroma
  • Infected Sebaceous Cyst

Most likely inguinal or femoral hernia that has become obstructed or strangulated.

Management:

  • IV fluid resuscitation with crystalloids
  • Analgesia (opioid)
  • NBM
  • Anti-emetics (as needed)
  • NG decompression
  • Fluid balance chart
  • Consider urinary catheter
  • Urgent surgical repair of the hernia and resection of infarcted/ necrotic bowel
    • Note: do not try to reduce a painful incarcerated hernia if it has been present for > 4 - 6 hours, otherwise the infarcted/ necrotic bowel may perforated intraperitoneally
73
Q
  1. This lady had a swelling on the back of her lower neck. It has been present for years. It had slowly enlarged over that time and is producing no other symptoms.
    1) What is the most likely diagnosis?
    2) what tests other than physical examination are likely to be helpful?
    3) What are the common complications of operative removal of this?
A

Most likely lipoma or epidermoid cyst

Tests:

  • FNA/cystology
  • USS for cystic lesions
  • MRI/CT best for lipoma to differentiate liposarcoma

Complications of removal

  • Infection
  • Seroma
  • Haemoatoma
  • Scarring
74
Q
  1. A 65 year old woman presents to the ED with vomiting, abdominal distention and colicky pain. she has never had an operation. She has a CT scan.
    1) what does this show?
    2) what does the surgical registrar do the minute he sees this scan?
    3) what does he find? what is the treatment?
A

Most likely sigmoid volvulus or incarcerated inguinal hernia

Sigmoid volvulus

  • Once sigmoid volvulus is diagnosed a sigmoidoscope is gently passed as far as possible into the rectum and a flatus tube inserted into the scope
  • The flatus tube is then gently manipulated through the twisted bowel
  • If successful there is a gush of liquid faeces and flatus, relieving the obstruction
  • The flatus tube is left in situ for 24 hours to maintain decompression, discourage re-twisting and allow recovery of the vascular supply to the bowel wall.
  • If the volvulus cannot be released, operative management is performed urgently
  • In most cases the bowel is still viable but sigmoid colectomy is usually required to prevent recurrence.
  • A double barrelled colostomy is generally safer than a primary anastomosis in a dilated and unprepared colon.

Incarcerated Inguinal hernia:

  • Once a hernia is seen, the registrar should attempt to reduce it, so long as the time since onset has been less than 4 – 6 hours
  • When reducing hernia’s it should be done slowly and gently, trying to reduce oedema first before reducing the contents back into the abdomen
  • Given the symptoms and signs of obstructed bowel the registrar will probably find the hernia irreducible.
  • Therefore management should be to treat the SBO obstruction and perform an emergency hernia repair +/- bowel resection, depending on whether the bowel is infarcted/ necrotic.
  • IV fluid resuscitation with crystalloids, NBM, analgesia (opioid), anti-emetic as required, NG decompression
75
Q
  1. What is the massive transfusion protocol? In what circumstances should it be used?
A

MTP is a standardized procedure for haemostatic resuscitation in critically bleeding patients. It should be utilized in all patients who are anticipated to require a massive transfusion, defined as:

  • Replacement of >1 blood volume in 24 hours
  • >50% of blood volume in 4 hrs
  • Transfusion of >40mL/kg in children

Goals:

  • Early recognition of blood loss
  • Maintenance of tissue perfusion and oxygenation by restoring blood volume and haemoglobin
  • Arrest of bleeding

Judicious use of blood component therapy to correct coagulopathy

76
Q
  1. A 75 year old lady was admitted with abdominal pain and vomiting, this X-ray was taken.
    1) What are the possible causes of her small bowel obstruction?
    2) What are the priorities in managing her in the emergency department?
A

Causes of Small Bowel Obstruction:

  • Most common
    • Postoperative adhesions
    • Hernias
  • Intrinsic to wall of small intestine
    • Tumours
    • Strictures (Crohns)
    • Intramural heamatoma
  • Causes of intraluminal obstruction
    • Intussusception
    • Gallstones
    • Foreign bodies

Management:

  • ABCDE approach
  • IV access x 2
    • Bloods: FBC/UEC/VBG/G roup + save
    • IVF
    • Analgesia
    • Anti-emetics
  • Early surgical consult
  • NG decompression
  • Nil by mouth
77
Q

127.

1) What is this structure and what operation is being attempted?
2) What symptoms would the patient have presented with?
3) What is the modern management of this condition?

A
78
Q
  1. How do you categorize causes of abdominal pain?
A

The surgical sieve** can be remembered via the mnemonic **VITAMIN CDEF”:

  • Vascular: Ruptured AAA, ischaemic bowel, ruptured ectopic pregnancy, testicular tosion, sickle cell crisis
  • Infective: appendicitis, cholecystitis, cholangitis, diverticulitis, pyelonephritis, PID, infective colitis
  • Trauma: Splenic, hepatic, renal or bowel injury
  • Autoimmune/ Inflammatory: IBD, Pancreatitis, peptic ulcer disease, mesenteric adenitis, coeliac disease
  • Metabolic: Renal failure with uraemia, ascites
  • Iatrogenic/ idiopathic: Medications, adhesions, incarcerated hernia
  • Neoplastic: Colonic/ gastric/ gynaecological/ renal malignancy, lymphoma
  • Congenital: Meckel’s diverticulum, indirect inguinal hernia
  • Degenerative: Radiculopathy from spinal pathology
  • Endocrine: Addisonian crisis, DKA, porphyria
  • Functional: IBS
79
Q
  1. Where is the appendix found?
A
  • The appendix is normally located in the RIF, as a blind ending pouch attached to the caecum.
  • The surface landmark for the appendix is at McBurney’s point, which is 1/3 of the way along a line drawn from the Anterior Superior Iliac Spine (ASIS) to umbilicus.
  • The 3 separate taenia coli converge on the appendix at the triradiated fold, commonly forming the ‘Mercedes Benz’ sign, with the appendiceal orifice in the middle, as seen on colonoscopy
  • Embryologically, the primordial caecum and appendix appears are a swelling on the antimesenteric border of the caudal limb of the midgut loop during the 6th week.
  • Initially the appendix is a small diverticulum of the caecum, however if progressively increases in length.
80
Q

When should you commence antibiotics in a patient who has symptoms and signs of appendicitis?

A
  • Antibiotic therapy should be commenced once surgical consult has been obtained and the surgeon makes a clinical diagnosis of appendicitis. Currently there is insufficient evidence to support antibiotic therapy as the mainstay of treatment for appendicitis.
  • Empirical therapy to cover gram-negative aerobes + anaerobes
    • Gentamicin
    • Ampicillin
    • Metronidazole
  • Once susceptibility results are obtained, antibiotics should be modified as appropriate.
  • If the appendix is not-perforated, only pre-operative Abx are required. Perforated appendixes mandate treatment with oral Abx postoperatively.
81
Q
  1. What are the important differential diagnoses to consider when managing a 17 year old woman who has the symptoms and signs of appendicitis?
A

Gynaecological:

  • Pelvis Inflammatory disease
  • Complications of an ovarian cyst (rupture, torsion)
  • Mittelschmerz from right ovary
  • Endometriosis
  • Ruptured ectopic pregnancy
  • Ovarian torsion
  • Uterine fibroids

Non-Gynaecological

  • Appendicitis
  • Gastroenteritis
  • Mesenteric adenitis
  • Inguinal hernia
  • Femoral hernia
  • Uteric colic
  • UTI (cystitis)
  • Pyelonephritis
  • Crohn’s disease
  • Meckel’s Diverticulitis
  • Caecal Volvulus
  • Cholecystitis
  • Small bowel obstruction
  • Pancreatitis
  • Perforated peptic ulcer
  • Constipation
82
Q
  1. You (the candidate) are an Emergency Department resident. How would you proceed to manage a 22 year old female patient who presents to you with a 24 hour history of Right Iliac Fossa Pain?
A

History

  • Pain: SOCRATES
  • Associated Symptoms
    • Haematuria, dysuria, urinary urgency
    • PR bleeding, diarrhoea, constipation, N&V
    • Fever and chills
    • Recent cold or flu
  • Gynae/Menstrual hx
    • LMP
    • Pregnant?
    • Previous Gynae Hx
    • Sexual hx
  • Meds/allergies
    • OCP use or other contraceptive
  • FHX
  • SHX

Examination

  • Vitals: HR, RR, BP, O2, Temperature
  • Abdominal Examination
    • Inspection (distension)
    • Palpation (masses, tenderness, peritonism)
    • Percussion (tenderness, peritonism, tympanic)
    • Auscultation (bowel sounds)
  • Pelvic examination (with chaperone)
    • PV bleeding/discharge
    • Inspection with speculum
    • Adnexal tenderness/masses
    • Cervical motion tenderness

Investigation

  • FBC, UEC, CRP, LFT
  • VBG
  • Goup and hold
  • Urine HCG and blood Beta-HCG
  • Urinalysis +/- urine MCS
  • Abdominopelvic USS

Management

  • IV fluid resuscitation with crystalloids
  • Analgesia (opioid)
  • Contact the surgical/gynaecology/gastroenterology registrar as appropriate
  • NBM
  • Consider Abx
83
Q
  1. What is the indication for imaging in suspected appendicitis?
A
  • Need to consider the balance between the need for diagnostic accuracy (reducing false positives proceeding to theatre) and preventing delay in timely definitive management.
  • Acute appendicitis is a clinical diagnosis, almost entirely made off history and examination
  • Imaging should only be used to exclude differential diagnoses and/ or confirm the diagnosis in atypical presentations
  • Plain film abdominal X-ray is only indicated to exclude other diagnoses:
    • Intestinal obstruction
    • Ruptured viscus
    • Suspected foreign body
    • Urolithiasis
  • USS abdomen/pelvis – due to the non-ionising nature it should be the investigation of choice in young patients (esp. in excluding gynae pathology)
  • CT abdomen/pelvis – has a very high sensitivity and specificity for the diagnosis of acute appendicitis but uses ionising radiation

USS findings in acute appendicitis

  • Aperistaltic, non-compressible, dilated appendix (>6 mm outer diameter)
  • Appendicolith
  • Distinct appendiceal wall layers
  • Echogenic prominent peri-caecal and peri-appendiceal fat
  • Peri-appendiceal hyperechoic structure: amorphous hyperechoic structure (usually >10 mm) seen surrounding a non-compressible appendix with a diameter >6 mm
  • Peri-appendiceal fluid collection
  • Target appearance (axial section)
  • Peri-appendiceal reactive nodal prominence/enlargement

CT findings in acute appendicitis

  • Dilated appendix with distended lumen (>6 mm diameter)
  • Thickened and enhancing wall
  • Thickening of the caecal apex (up to 80%)
  • Peri-appendiceal inflammation, including stranding of the adjacent fat and thickening of the lateroconal fascia or meso-appendix
  • Extraluminal fluid
  • inflammatory phlegmon
  • Abscess formation
  • Appendicolith
  • Peri-appendiceal reactive nodal prominence/enlargement
84
Q
  1. Why does the pain and discomfort of appendicitis classically ‘migrate’ from generalised abdominal pain to become localised in the RIF?
A

Shift from visceral to somatic pain.

  • There are 2 pain sensation systems in the abdomen:
    • The splanchnic (visceral) system, which only senses stretch and spasm
    • The spinocerebral (somatic) system that can sense the same array of painful stimuli as skin.
  • The embryonic gut arises as a midline organ and its splanchnic innervation is bilateral, thus visceral pain is felt in the midline.
  • The splanchnic nerves carrying this information synapse on neurons which also receive inputs from the anterior abdominal wall – the brain misinterprets this visceral signal as arising from the anterior abdomen.
  • The appendix, being an embryological midgut organ, is innervated by the lesser splanchnic nerve (T10-T11) and thus initial pain is felt around the umbilicus.
  • There is no somatic innervation of the visceral peritoneum, but as the appendix progressively distends and the inflammatory process extends to involve the parietal peritoneum, the pain signals are carried in somatic nerves and inflammation can be localised to the actual site of the appendix – usually in the RIF.

This shift in pain location usually occurs in less than 24 hours

85
Q
  1. What are the indications for a laparscopic approach to appendicetomy?
A
  • Uncertain diagnosis
    • Allows inspection of other organs e.g. pelvic pathology
  • Obese patients
    • Exposure of R lower quadrant requires larger incision in obese patients
  • Elderly patients
    • Shorter hospital stay
    • Discharge to home rates higher
  • Laparoscopy à shorter recovery and decreased length of hospital stay, increased theatre costs and operative time
    • Fewer SSIs
    • More intra-abdominal abscesses
86
Q
  1. What are the classical symptoms and signs of acute appendicitis?
A
  • The classical picture is seen in less than half of all cases of appendicitis, largely due to the anatomical variation of the appendix and the vigour of the body’s defences

Classical Symptoms:

  • Classical appendicitis begins with poorly localised colicky central abdominal visceral pain (midgut pain); resulting from smooth muscle spasm as a reaction to appendiceal obstruction. The patient also has anorexia and nausea/ vomiting at this stage.
  • As the inflammation advances it progresses through the appendiceal wall to involve the parietal peritoneum, which is innervated somatically.
  • The pain then becomes localised to the RIF.

Classical Signs:

  • Signs of localised peritonitis can be elicited:
    • Tenderness in the RIF
    • Guarding
    • Rebound tenderness
  • Low grade fever
  • Tachycardia
  • Diminished bowel sounds

Other signs:

  • Obturator sign (pain with passive flexion and internal rotation R hip)
  • Psoas sign (pt lies on left side, pain with passive extension R hip)
  • Rovsing’s sign (pain in RLQ with palpation in LLQ)
87
Q
  1. What is a sentinel node biopsy in the management of breast cancer?
A
  • Sentinel Node Biopsy is one of the 3 methods for axillary staging of breast cancer (the other 2 are axillary sampling or clearance)
  • This techniques has become the standard of care in the management of the axilla in patients with clinically node-negative disease
  • Lymphatic spread nearly always follows a predictable pattern.
  • The aim is to identify the first node which drains the breast (and is thus the most likely site of metastasis)
  • The node is identified by both injection of a radioactive isotope bound to albumin (injected next to the tumour in the same quadrant, 12 hours before surgery) and a blue dye injected into the periareolar area (subdermal plexus) of the same quadrant at surgery.
  • The sentinel node is located during surgery with a gamma probe (Geiger counter) and then surgically visualised by the blue dye.
    • The sentinel node is located low in the axilla in >90% of cases
  • The blue nodes are excised and examined by a pathologist – this can be done by immediate frozen section, and if the nodes are positive, an axillary clearance can be completed
    • The main advantage on SNB is considerably increased sensitivity of lymph node analysis, and avoidance of the high morbidity (such as lymphoedema, long thoracic nerve injury, axillary vein thrombosis, etc.) associated with axillary clearance
88
Q
  1. What is a hernia?
A

A hernia is the abnormal protrusion of a viscus (organ) through the wall of the cavity in which the viscus is normally contained.

89
Q
  1. What is the anatomical difference between an inguinal hernia and a femoral hernia?
A
  • Inguinal hernias arise from above the inguinal ligament and femoral hernias from below
  • Inguinal hernias can be either direct or indirect

Indirect inguinal hernia:

  • The hernia sac enters the inguinal canal at the deep (internal) inguinal ring
    • Located lateral to the inferior epigastric artery at the midpoint of the inguinal ligament
    • This is where the spermatic cord exits the abdomen in males
    • This is where the round ligament of the uterus exits the abdomen in females
  • The hernia may then pass inferomedially to emerge via the superficial (external) ring, which may extend into:
    • Scrotum in males
    • Labia majora in females

Direct inguinal hernia:

  • The hernia sac directly protrudes through the anterior abdominal wall (Hesselbach’s triangle)
    • This is located medial to the inferior epigastric artery
  • These hernias rarely extend into the scrotum due to there being no guiding path

Femoral hernia:

  • Project through the femoral canal, which is located below the inguinal ligament

Inguinal hernias

Indirect

  • Lateral to inferior epigastric artery
  • Usually congenital, due to incomplete obliteration of the processes vaginalis
  • Exit via the deep inguinal ring (in transverse abdominus) in to the inguinal canal
    • Borders:
      • Anterior: External oblique aponeurosis
      • Inferior: Inguinal ligament
      • Superior: Transverse abdominus muscle
      • Posterior: Transverse fascialis
  • Occur in men and women – round ligament of the uterus takes same path through inguinal canal to labium majora

Direct

  • Protrusion through transverse fascia within hesselbach’s triangle
  • Medial: Rectus abdominus
  • Lateral: Inferior epigastric artery
  • Inferior: Inguinal ligament

Femoral hernias

  • Project through the femoral canal, neck is below inguinal ligament
  • Borders of femoral canal:
    • Anterior: Inguinal ligament
    • Posterior: Pectineus muscle
    • Lateral: Femoral vein
    • Medial: Lacunar ligament
90
Q

Extra: What are the anatomical boundaries of

a) Inguinal Canal
b) Hasselbach’s Triangle
c) Femoral canal

A
91
Q
  1. On examination, what is the difference between an inguinal hernia and femoral hernia?
A

Examination of a hernia

  • Patient standing à ask patient to point to site of lump
  • Inspect for lumps, swelling, scars
  • Ask patient to cough and inspect for cough impulse in region of pubic tubercle
  • Palpate over pubic tubercle and ask patient to cough again à palpate expansile impulse
  • Repeat with patient supine
  • Identify type of hernia:
    • Femoral hernias
      • Smaller and firmer
      • Neck is below inguinal ligament à palpate femoral artery and move medially
      • Frequently unreducible, rarely have cough impulse
    • Inguinal hernias
      • Above inguinal ligament
      • Invagination of scrotum in external ring may reveal indirect hernia
      • Larger
92
Q
  1. What is the importance of deciding whether a hernia is inguinal or femoral?
A
  • Femoral hernias have a higher rate of complications than inguinal hernias à more likely to become incarcerated/strangulated
  • Asymptomatic inguinal hernias may be managed by a watch and wait approach, whereas femoral hernias should be repaired without delay –> better done electively than emergently
  • Determine surgical approach and operation performed
    • Approach for femoral hernia repair:
      • Femoral (low)
      • Lotheissen (high)
      • Posterior wall of inguinal canal
    • Approach for inguinal hernia repair
      • Bassini – used sutures
      • Lichtenstein – uses mesh
93
Q
  1. What is the difference between an incarcerated or irreducible hernia, and a strangulated hernia?
A
  • A reducible hernia can be pushed back into the right place
  • An irreducible hernia cannot be pushed back into the right place. This is not a problem for the patient per se but is at risk of causing problems. A hernia may be irreducible because it is:
    • Incarcerated: stuck in the abnormal position because either there are adhesions between the hernial sac and surrounding structures, or there are adhesions between structures within the sac such that the sac is now wider than its neck and no longer able to pass through the wall defect
    • Obstructed: the neck of the hernial sac provides an obstruction to the passage of flow through the bowel lumen (but the blood supply of the bowel is still intact)
    • Strangulated: if the bowel becomes so constricted by the neck of the hernial sac that the blood supply is compromised it is referred to as a strangulated hernia. This is a major problem for the patient as their bowel will necrose rapidly from ischaemia.
94
Q
  1. A 75 yoa woman presents to the emergency department with a 24 hour history of vomiting and cramping abdominal pain. She has not passed flatus or stool for 24 hours. She has no relevant previous medical history. On examination, she has a distended abdomen, and a tender red coloured lump in the right groin. What is the most likely diagnosis and what should be done to help this patient?
A

Most likely: bowel obstruction due to strangulated hernia (inguinal/femoral). Femoral hernias are more common iin women and more likely to strangulate, however the overall incidence of inguinal hernias is higher

Differentials:

  • Inguinal hernia
  • Femoral hernia
  • Reactive lymphadenitis
  • Femoral aneurysm
  • Psoas abscess/psoas bursitis
  • Saphena varix/ valvular varicosity

Management

  • Arrange general surgical consult
  • Insert 2 x large bore IV cannulas
    • Bloods – FBC / UEC / LFT / Group and save/ Coags / ABG
    • IV fluids
    • Analgesia
  • Anti-emetics
  • NG tube for bowel decompression
  • Urinary catheter
  • Monitor fluid balance
  • Open repair à do not use mesh if strangulated due to risk of infection.
95
Q
  1. What are the common causes of a lower GI bleed?
A
  • Lower GI bleed: distal to the ligament of Trietz (duodenal-jejunal flexure)

Common causes by ages:

  • All ages: haemorrhoids
  • Adults: colonic diverticulitis and angiodysplasia
  • Older patients with CVD: ischemic colitis
  • Children: anorectal fissures, infectious colitis

Causes by systems:

  • Anatomic
    • Diverticulosis
  • Vascular
    • Angiodysplasia
    • Ischaemic
    • Radiation-induced
  • Inflammatory
    • Infectious
    • IBD
  • Neoplastic
  • Iatrogenic
96
Q
  1. What are the signs and symptoms of a lower GI bleed?
A
  • Haematochezia (maroon or bright red blood PR)
  • Melaena
  • Tachycardia
  • Hypotension (may be orthostatic)
  • Dizziness/ light-headedness
  • Chest pain/ SOB/ palpitation (due to anaemia)
  • Associated Symptoms:
    • Abdominal pain/ perianal pain
    • Change in bowel habits & Weight loss à malignancy
    • Night sweats
    • Fevers
    • Abdominal masses
  • Acute bleeding –> normocytic anaemia
  • Chronic bleeding -> microcytic anaemia
97
Q
  1. What is the management fo a large LGI bleed?
A

Summary:

  • Initial assessment and resuscitation
  • Exclusion of UGI bleeding
  • Evaluation of source of LGI bleeding
  • Definitive management of the cause

Initial Assessment – ABCDE:

  • Assess vitals (HR, RR, BP, SaO2, Temperature)
  • Give supplemental oxygen if hypoxic
  • Gain IV access (2x Large Bore IV cannulas) and resuscitate with IV crystalloids
  • Alert ICU/ HDU if required
  • Bloods: VBG, FBC, UEC, Coagulation studies, LFT, CRP, Group and Hold (or X-match)
  • Consider the need for blood products to replace lost blood and correct for coagulopathy or thrombocytopaenia
  • Consider an IDC to monitor urine output
  • Keep patient NBM
  • NGT lavage to exclude UGI source of bleeding if moderate to low suspicion of UGI cause (proceed to endoscopy if high suspicion of UGI causes)
  • Early contact with gastroenterology/ anaesthetics/ general surgery as appropriate

History:

  • Age
  • Duration of bleeding
  • How much blood? Haematemesis?
  • Colour of blood (haematochezia/ melaena), any mucous?
  • Associated dizziness or syncope
  • Other symptoms – pain (abdominal/ rectal), constipation/ diarrhoea, tenesmus, change in bowel habits, weight loss
  • PMHx – IBD, haemorrhoids, diverticular disease, Angiodysplasia, past colonoscopies? AAA repair?
  • Meds – aspirin/ NSAIDs, anticoagulants, recent Abx
  • Allergies
  • Recent travel
  • EtOH Hx
  • When was last meal

Examination:

  • Vitals + postural hypotension
  • Abdominal examination – tenderness, organomegaly, masses
  • PR exam – rectal mass, assess colour of blood/ stool

Investigation/ Definitive management:

  • Bloods as listed above
  • Stool MCS (ova, cysts and parasites if associated with diarrhoea)
  • Anoscopy – to exclude simple anorectal causes
  • Colonoscopy – requires bowel prep, bleeding can be controlled with cautery, injection of adrenaline, band ligation, haemoclips
  • Radiographic studies:
    • Radionuclide imaging (Technitium-99m labelled colloid or RBCs)
    • CT angiography
    • Mesenteric angiography (with embolisation)
  • Emergency Laparotomy
98
Q
  1. What is a Meckel’s Diverticulum?
A
  • Most common congenital anomaly of the gastrointestinal tract.
  • True diverticulum
  • Formed by persistent proximal part of vitellointestinal duct (4th week) between foregut and yolk sac.
  • Also known as omphalomesenteric gut
  • During 9th week this duct normally gets obliterated.
  • If whole duct remains –> Vitellointestinal fistula (between foregut and anterior abdominal wall)
  • If proximal remains –> Meckels diverticulum

Rule of 2’s

  • Incidence: 2%
  • 2% of cases become symptomatic
  • Location: 2 ft from ileocecal junction 147
  • Length: 2 inches
  • Age of presentation: 2 years
  • M:F – 1:2
  • 2/3rd have ectopic mucosa
  • Two types of ectopic tissue: gastric and pancreatic ->pleuripotent Cells -> gastric tissue -> acid secretion -> ulceration

Diagnosis

  • Technitum -99 Pertechnate scan – meckels scan
  • Detects gastric tissue
  • Other tests: USG
  • Ct scan

Tx: if symptomatic surgical resection

99
Q
  1. What symptoms can be caused by a Meckel’s Diverticulum and why?
A

most patients remain asymptomatic

  • PR Bleeding (frank blood or occult blood):
    • Most common complication
    • Acid secreting gastric mucosa causes inflammation and ulceration of adjacent small bowel wall
  • Obstructive Symptoms (abdominal distension, N/V, cramping periumbilical pain):
    • 2nd most common complication
    • SBO due to adhesions, volvulus, intussusception (if inverted), herniation, stricture formation, neoplastic obstruction
  • Abdominal Pain (epigastric migrating to the RIF):
    • Meckel diverticulitis due to a narrow neck and inflammation – mimic of appendicitis
  • Peritonism:
    • Due to perforation of the diverticulum
100
Q
  1. What are the causes of an obstruction of a luminal structure? (e.g. the bowel, ureter, blood vessel)
A

Intraluminal, intramural, extramural and functional

Intraluminal:

  • Impacted faeces
  • Tumour
  • Gall stones
  • Renal calculi
  • Thrombus/embolus

Intramural

  • Malignancy
  • Strictures
  • Intussusception
  • Atherosclerosis

Extramural

  • Adhesions
  • Congenital bands
  • Tumours
  • Hernia’s
  • Volvulus
  • Prostatic hypertrophy
  • External compression
  • Mirizzi syndrome (gallstones)

Function/pseudo-obstruction:

  • Post-op ileus
  • Electrolyte abnormality
  • Medication-induced
101
Q
  1. What are the symptoms of an upper GI bleed?
A

Bleeding manifestations

  • Nausea and vomiting – haematemesis
    • Oxidation of iron causes coffee ground appearance
  • Melaena
  • Haematochezia (brisk UGIB)

Signs of hypovolaemia

  • Dizziness/lightheadedness
  • Chest pain
  • Weakness and fatigue
  • Tachycardia
  • Thready pulse
  • Tachypnoea
  • Dyspnoea
  • Orthostatic hypotension à supine hypotension
  • Delayed capillary refill
  • Cool extremities
  • Oliguria

Site specific

  • Peptic ulcer à epigastric/LUQ pain
  • Oesophageal cancer: odynophonia, GORD, dysphagia
  • Mallory-weiss tear: emesis, retching, coughing prior to haemoptysis
  • Variceal haemorrhage/ portal hypertensive gastropathy: jaundice/ weakness/ fatigue / anorexia / abdominal distension / spider naevi
  • Malignancy: dysphagia/ early satiety/ involuntary weight loss / cachexia
102
Q
  1. What is the treamtent of an upper GI bleed?
A

ABCDE approach

  • Assess vitals (HR, RR, BP, SaO2, Temperature)
  • Give supplemental oxygen if hypoxic
  • Gain IV access (2x Large Bore IV cannulas) and resuscitate with IV crystalloids
  • Alert ICU/ HDU if required
  • Bloods: VBG, FBC, UEC, Coagulation studies, LFT, CRP, Group and Hold (or X-match)
  • Consider the need for blood products to replace lost blood and correct for coagulopathy or thrombocytopaenia
  • Consider an IDC to monitor urine output
  • Keep patient NBM
  • Consider placement of NGT
  • Early contact with General surgery/ anaesthetics/ gastroenterology
  • IV pantoprazole (PPI)
  • Erythromycin (prokinetic to help with endoscopic visualisation)

History and Examination

  • N/V/D
  • Haematemesis/ haematochezia/ melaena
  • Duration of symptoms
  • Relationship of bleeding to vomiting
  • Dietary causes – i.e. red food/ beverages
  • Constitutional symptoms
  • Associated symptoms: abdo pain?
  • PMHx: PUD, varices, alcoholism, chronic liver disease, haematological disease, GORD/ Oesophagitis, AAA repair
  • Meds: Aspirin/ NSAIDs, anticoagulants
  • EtOH and smoking
  • Exam – anaemia, hypovolaemia (+ orthostatic hypotension), vitals, signs of liver disease, abdominal examination + PR exam

Definitive Management:

  • Endoscopy (injection of adrenaline, cautery, banding, haemoclips)
  • Emergency laparotomy if endoscopic treatment fails

Risk Stratification:

  • NICE guidelines recommend the use of the GBS at first assessment and a full Rockall score after endoscopy
  • GBS - A score of 6 or more on GBS is a >50% likelihood of needing intervention
  • Rockall – a score of < 3 carries a good prognosis, but a total score of 8 or higher carries a high risk of mortality
103
Q
  1. What do you do as the intern if you think someone is having an acute UGI bleed?
A

Call for help early

Initial assessment: ABCDE

  • Assess vitals (HR, BP, RR, Temp, SaO2)
  • Is the patient shocked?
  • Call for help early (ED/ ICU or MET)
  • Give supplemental oxygen if hypoxic
  • Gain IV access (2x large bore IV cannulas) and resuscitate with IV crystalloids
  • Initial investigations:
    • Bloods: VBG, FBC, UEC, LFT, Coagulations studies, CRP, group and hold (or X-match)
    • ECG and ?CXR
  • Consider the need for blood products to replace lost blood and correct for coagulopathy or thrombocytopaenia
  • Consider an IDC to monitor urine output
  • Keep patient NBM
  • Consider placement of NGT

History and examination

  • N/V/D
  • Haematemesis/ haematochezia/ melaena
  • Duration of symptoms
  • Relationship of bleeding to vomiting
  • Dietary causes – i.e. red food/ beverages
  • Constitutional symptoms
  • Associated symptoms: abdo pain?
  • PMHx: PUD, varices, alcoholism, chronic liver disease, haematological disease, GORD/ Oesophagitis, AAA repair
  • Meds: Aspirin/ NSAIDs, anticoagulants
  • EtOH and smoking
  • Exam – anaemia, hypovolaemia (+ orthostatic hypotension), vitals, signs of liver disease, abdominal examination + PR exam
104
Q
  1. What are the risk factors contributing to the aetiology of a peptic ulcer?
A

A peptic ulcer is a defect in the mucosal lining of the stomach or duodenum that reaches the depth of the submucosa (smaller defects are called erosions)

  • H. Pylori infection
  • NSAIDs: Dose/ duration of action/ duration of therapy/ advanced age
    • Co-therapy of drugs increasing toxicity à steroids, low dose aspirin, SSRIs, alendronate
  • Prior hx of PUD
  • Smoking à increased vagal activity, gastric mucosal ischemia
  • Overproduction of gastrin (Zollinger-Ellison Syndrome)
  • Increasing age
  • HFx
  • Gastrinoma
  • Crohn’s
  • Sarcoidosis
  • CMV, herpes simplex, tuberculosis
  • Critical illness à burns, head injury, physical trauma, multiple organ failure
  • Increasing age
105
Q
  1. What are the inital investigations in the ED for a perforated ulcer?
A
  • Following Initial assessment with history, physical examination and resuscitation (as required).

Initial Investigations:

  • Bloods:
    • VBG
    • FBC, UEC, LFT, Coags, CRP, Group and hold, lipase, blood cultures
  • Radiology:
    • Erect CXR (pneumoperitoneum)
    • CT abdo/pelvis may be used
  • Other:
    • ECG
106
Q
  1. What are the physical signs suggestive of a perforated ulcer?
A

Signs of perforated PUD:

  • Involuntary guarding/ “board-like” rigidity
  • Generalised abdominal tenderness
  • Rebound/ percussion tenderness
  • Absent bowel sounds
  • Maybe abdominal distension and diffuse tympany
  • Signs of shock:
    • Cold and sweating with rapid shallow breathing
    • Tachycardia with thready pulse
    • Hypotensive
  • Febrile

Perforated Ulcer Natural Hx

Phases:

  • Initial (0-2hrs)
    • Perforation results in bathing of the peritoneal cavity with acidic fluid, which likely results in the release of vasoactive mediators that underlie the physiological response.
    • Abdominal pain sudden
      • Epigastric but becomes generalised
    • Tachycardia
    • Weak pulse
    • Cool extremities
    • Low temperature
  • Second phase (2-12hrs)
    • Abdo pain may lesson
    • Pain generalized, worse with movement, abdomen consistently displays marked board-like rigidity.
    • PR examination of pelvic peritoneum - tender due to irritation from collected inflammatory fluid
  • Third phase (>12hrs)
    • Increasing abdominal distension is noted
    • Abdo pain, tenderness and rigidity less evident.
    • “Third-spacing” into the peritoneal cavity and acute cardiovascular collapse
      • Temperature elevation
      • Hypovolemia
107
Q
  1. What are the symptoms suggestive of a perforated peptic ulcer?
A
  • Shallow breathing
  • Febrile once secondary bacterial peritonitis occurs
  • Rigidity -> involuntary spasm
  • Epigastric tenderness -> may not be marked due to guarding
  • Tympanitic liver due to underlying pneumoperitoneum
  • Abdominal distension and diffuse tympany -> advanced
  • Signs of shock
    • Tachycardia
    • Thready pulse
    • Cool extremities
    • Hypothermia
  • Pain on PR due to irritation of pelvic peritoneum
108
Q
  1. What is the treatment for a perforated duodenal ulcer?
A

Initial:

  • ABCDE
  • Supplemental O2
  • IV access
    • Crystalloid resuscitation
    • Blood: VBG, FBC, UEC, LFT, Coags, Group and Hold (X match)
    • Transfusion of blood products if required
  • NBM
  • Analgesia (opioid)
  • Consider IDC
  • Consider NGT
  • Establish Dx: erect CXR/Ct
  • IV PPI (pantoprazole)
  • Empirical Abx (triple therapy – ampicillin, gentamicin, metronidazole)
  • General surgical consult

Surgical management

  • Laparotomy or laparoscopy
  • Inspection of the abdomen and diagnosis confirmed
  • Duodenal perforation is repaired with simple closure by suturing a flap of omentum over the defect (“Graham patch”) – followed by H. Pylori eradication therapy
  • Peritoneal toilet

Conservative management

  • Stop NSAIDs
  • Triple Therapy for H. Pylori
    • Omepreazole, Clarithromycin, amoxycillin
  • Quadruple therapy
    • PPI (omeprazole/lansoprazole) + bismuth + metronidazole + tetracycline
  • Confirm eradication
109
Q
  1. How do you (as a surgical or ED intern) get a patient with a perforated ulcer ready for theatre?
A
  • Adequate resuscitation – IV access and chart fluids
  • Oxygen supplementation if hypoxic
  • Know significant Hx, Exam and Ix results
  • Keep the patient NBM
  • Adequate analgesia (opioid)
  • Consult from general surgical team
  • Consult from anaesthetic team
  • Contact ICU and get them involved
  • Contact the blood bank and ensure group and hold or X-match is done
  • IDC
  • NGT
  • IV PPI
  • IV Abx (triple therapy – ampicillin, gentamicin, metronidazole)
  • Correct electrolyte abnormalities (if possible)
  • Correct coagulation abnormalities (if possible)
  • Have the surgical registrar consent the patient
  • Deliver OT booking slip to OT NUM
110
Q
  1. You are the general surgery resident on call, you are asked to review a 70 year old man who is 72 hours post knee replacement on the orthopaedic ward. He has abdominal pain and distension, he has not been passed stool or wind since the first day post surgery, he started vomiting this evening. He is usually well, with no chronic health problems found in his pre-surgery investigations.
A
  • Most Likely = Post-operative ileus or Acute Colonic Pseudo-obstruction (ACPO) a.k.a. Ogilvie Syndrome – acute dilatation of the colon in the absence of mechanical obstruction

Initial Management:

  • ABCDE (check vitals)
  • Have the patient NBM
  • Give IV crystalloid fluid resuscitation
  • Analgesia (non-opioid)
  • History:
    • Check the operation report
    • SOCRATES of pain
    • Diet
    • Review the notes and medication chart
  • Examination:
    • Surgical wound
    • Systemic symptoms/ signs
    • Abdominal exam, including PR exam

Investigations:

  • Bloods: FBC, UEC, CMP, LFT, VBG, Lipase
  • Imaging: Erect CXR and supine AXR, CT abdomen

Management:

  • Serial abdominal examination
  • Cease non necessary medications that decrease colonic motility
  • Prokinetic: metoclopramide, erythromycin
  • Analgesia (non-opioid)
  • NGT for decompression
  • Encourage ambulation
  • Neostigmine (acetylcholinesterase inhibitor) if caecal dilatation > 12cm
  • If considered opioid induced then give naltrexone
  • If no response the patient may need decompression (non-surgical or surgical)
111
Q
  1. What are the common skin cancers?
A

Malginant:

  • basal cell carcinoma (BCC)
  • Squamous cell carcinoma (SCC)
  • Melanoma

Pre-cancerous:

  • solar keratoses (actinic keratoses)
  • bowen’s disease
  • dysplastic naevus syndrome
  • giant congenital melanocytic naevi

Benign:

  • melanocytic naevi (moles)
  • basal cell papilloma (Seborrhoeic wart)
  • epidermoid cyst (sebacous cyst)
  • keratoacanthoma
112
Q
  1. What features of a mole on the skin would make you concerned that it had become malignant?
A
113
Q
  1. What are the principles of initial treatment of a mole on the skin that you are concerned may have undergone malignant change?
A

Investigation:

  • Histological confirmation with biopsy – total excision with a 3 -4mm margin, shave biopsy, punch biopsy (if suspicious of melanoma perform total excision biopsy)
  • SLN biopsy should be discussed with patients that have melanoma with a Breslow thickness > 1mm, however it does not alter survival prospect, rather it provides accuracy to prognosis

Management:

  • Follow up with the patient after pathological results are available
  • Wide local excision to appropriate margins if not already achieve
  • Staging (if required)
114
Q
  1. What is a volvulus of the colon?
A
  • A condition in which the bowel becomes twisted on its mesentery, causing partial or complete obstruction and vascular compromise
    • The sigmoid colon is the most commonly affected due to its long mesentery
    • The second most common is the caecum
  • Clinical Presentation:
    • Abdominal distension
    • Variable abdominal pain (rarely tenderness)
    • Obstipation
    • Tympanic abdomen
    • Nausea (maybe vomiting)
    • If infarct/ necrosis:
      • Severe pain, shock, tachycardia, peritonism/ rebound tenderness, fever
  • Pathogenesis:
    • Patients have a longstanding history of chronic constipation, which causes an elongated and atonic colon – acquired/ idiopathic megacolon
    • Occasionally a huge sigmoid loop loaded with faeces and gas becomes twisted on the mesenteric pedicle and produces a closed loop obstruction.
    • The obstruction leads to venous congestion, oedema, strangulation, gangrene and eventually perforation with faeculent peritonitis
115
Q
  1. How is a sigmoid volvulus treated (after initial history, examination and investigation)?
A

A patient presenting with presumed bowel obstruction:

  • IV fluid resuscitation with crystalloids
  • Analgesia
  • NBM
  • Anti-emetics (if needed)
  • NG decompression (if needed)

Once sigmoid volvulus is diagnosed:

  • A sigmoidoscope is gently passed as far as possible into the rectum and a flatus tube inserted into the scope
  • The flatus tube is then gently manipulated through the twisted bowel
  • If successful there is a gush of liquid faeces and flatus, relieving the obstruction
  • The flatus tube is left in situ for 24hrs to maintain decompression, discourage re-twisting and allow recovery of the vascular supply to the bowel wall.
  • If the volvulus cannot be released, operative management is performed urgently
  • In most cases the bowel is still viable but sigmoid colectomy is usually required to prevent recurrence.
  • A double barrelled colostomy is generally safer than a primary anastomosis in a dilated and unprepared colon.
116
Q
  1. This 18-year-old man has this lump in his groin. What is the most likely diagnosis? What are the differential diagnoses?
A

most likely: indirect inguinal hernia

117
Q
  1. This 18-year old man has this lump in his groin. It’s a hernia. What is the most likley to be in the hernia?
A

Contents of hernia’s

  • Preperitoneal fat
  • Peritoneal fat (e.g. mesenteric fat, omental fat etc.)
  • Small Bowel loops
  • Mobile large bowel (sigmoid, caecum, appendix)
  • Rarely other structures – bladder etc.
  • Most direct hernias do not have a true peritoneal lining (i.e. they have a pseudo-sac formed from the transversalis fascia) and do not contain bowel. They usually contain preperitoneal fat and occasionally bladder
  • Indirect hernias have a peritoneal sac (i.e. patent processus vaginalis) and most commonly contain mesenteric fat but may also contain small bowel loops or mobile colon segments (sigmoid, caecum, appendix)
  • Femoral hernias also have a protrusion of peritoneal sac and may contain pre-peritoneal fat, omentum or mesenteric fat, small bowel and rarely other structures.
118
Q
  1. What are common methods of repairing an inguinal hernia in a fit 18 year old?
A
  • Watchful waiting with eventual elective repair
    • Whether all hernias should be repaired or not is controversial
    • Indications for repair:
      • Strangulation/ incarceration
      • Symptoms – pain on lifting/ exertion, interference with ADLs
      • Femoral Hernia
  • Surgical Management is the definitive treatment
    • Open repair – may or may not use mesh (i.e. tension free mesh repair vs. primary tissue approximation repair)
      • Open repair is more common than laparoscopic repair for inguinal hernias
      • However, the laparoscopic approach is preferred for bilateral, recurrent and femoral hernias
  • Laparoscopic mesh repair
    • Total Extraperitoneal Surgery (TEPS) – most common laparoscopic method
  • Space developed between the peritoneum and the anterior abdominal wall
    • Trans-Abdominal Pre-Peritoneal Surgery (TAPS)
      • The peritoneal cavity is entered, and the mesh

Principles of hernia repair

  • Correct aggravating factors e.g. chronic cough/ prostatic obstruction/ colonic tumour/ ascites
  • Definitive treatment for all hernias is surgical repair
  • Indirect hernia sac à isolate, dissect to origin from peritoneum and ligate, reduce internal ring to adequate size
    • Repair inguinal floor in adult hernias (most likely not needed for 18 y/o pt)
  • Primary repair of direct hernia usually not appropriate due to intrinsic weakness of floor of inguinal canal
  • If direct inguinal hernia found à always search spermatic cord for indirect hernia
  • Goal of surgery is to reduce contents of hernia into abdomen and close fascial defect in inguinal floor
  • Inguinal hernia repair recommended if moderate to severe symptoms, otherwise can be managed with watchful waiting:
    • Groin pain with exertion
    • Inability to perform daily activities due to pain/ discomfort
    • Inability to manually reduce hernia (chronic incarceration)

Open repair methods

Tension-free mesh repair

  • Studies demonstrate sue of mesh facilitates faster recovery, reduces postoperative groin pain and reduces recurrence rate
  • Lichtenstein
    • Incise skin over inguinal canal
    • Divide subcutaneous layer, ligate superficial epigastric vein
    • Dissect subcutaneous tissue from external oblique aponeurosis to expose external inguinal ring
    • Incise aponeurosis of external oblique muscle extending laterally from external inguinal ring
      • Exposes internal oblique muscle à identify ilioinguinal nerve between internal and external oblique muscles
    • Dissect spermatic cord in Hesselbach’s triangle, retract -> in women, remove segment of round ligament
    • Resect peritoneal sac and repair internal inguinal ring or reduce in to preperitoneal space
    • Cut polypropylene mesh to cover inguinal region -> must tailor to patients anatomy
    • Suture mesh over inguinal ligament
  • Plug and patch
    • Hernia sac freed and inverted
    • Mesh placed over hernia defect, flat piece of mesh placed over inguinal floor
  • Kugel (preperitoneal repair)

Open primary tissue approximation

  • Used when mesh is contra-indicated:
    • Active infection/ contamination of groin
    • Cost-prohibitive setting
  • Shouldice
    • Three layers of inguinal canal dissected
    • Hernia reduced
    • Reconstruction of inguinal canal with four layer overlap technique with non-absorbable sutures
  • Bassini
    • Primary tissue approximation approach
    • Inguinal floor strengthened by suturing conjoined tendon to inguinal ligament form pubic tubercle medially to area of internal ring laterally
  • Desarda -> repairs floor of inguinal canal with external oblique muscle aponeurosis

Laparoscopic Techniques

  • Posterior approach of hernia defect
  • Totally extraperitoneal (TEP) repair
    • Space developed between peritoneum and anterior abdominal wall
  • Transabdominal preperitoneal patch (TAPP)
    • Mesh placed in preperitoneal position à peritoneum keeps mesh away from bowel.
119
Q
  1. What are the common complications of an inguinal hernia repair, and what are the serious complications of this operation?
A

Short term complications:

  • Common
    • Scrotal haematoma
    • Wound infection
    • Wound seroma
  • Serious
    • Urinary retention post-operatively
    • Division of the vas deferens
    • Visceral and vascular injury
    • Ischaemic orchitis post-surgery
    • Incision or port site hernia
    • Bowel obstruction post-surgery
    • Bladder injury

Long term complications

  • Recurrence
    • Inappropriate technique – Bassini repairs (25%)
    • Operator inexperience
    • Technical failure
    • Missed diagnosis of concomitant femoral hernia
    • Underlying physiological problems
  • Chronic groin pain due to inadvertent trapping of the ilioinguinal or other nerve in the repair
  • Testicular atrophy (due to damage of the testicular artery by diathermy or overtightening the deep ring.
  • Mesh rejection/migration/erosion
  • Pelvic adhesions
120
Q
  1. What is the most significant abnormality in this photo (the photo is a lump in the neck of a 70 yoa man). where, anatomically is it, what is the differential diagnosis?
A
  • The anterior and posterior triangles are divided by SCM
  • Anterior triangle boundaries – Inferior ramus of the mandible, anterior border of SCM, and the midline of the anterior neck
    • Structures under SCM are also considered part of the anterior triangle
  • Posterior triangle boundaries – posterior border of SCM, superior border of the clavicle, and the anterior border of trapezius
  • Anterior divisions:
    • Submental: anterior belly of digastric, body of hyoid bone, midline
    • Submandibular (digastric): anterior and posterior belly of digastric, mandible
    • Carotid: SCM, posterior belly of digastric, superior belly of omohyoid
    • Muscular: SCM, superior belly of omohyoid, midline from hyoid to jugular notch
  • Posterior divisions:
    • Supraclavicular: posterior border of SCM, inferior belly of omohyoid, clavicle
    • Occipital: posterior border of SCM, inferior border of omohyoid, anterior border of trapezius
121
Q

169 extra: anatomical triangles of the neck

A
  • Anterior divisions:
    • Submental: anterior belly of digastric, body of hyoid bone, midline
    • Submandibular (digastric): anterior and posterior belly of digastric, mandible
    • Carotid: SCM, posterior belly of digastric, superior belly of omohyoid
    • Muscular: SCM, superior belly of omohyoid, midline from hyoid to jugular notch
  • Posterior divisions:
    • Supraclavicular: posterior border of SCM, inferior belly of omohyoid, clavicle
    • Occipital: posterior border of SCM, inferior border of omohyoid, anterior border of trapezius
122
Q
  1. What specific hisotry and examination will you perform for a 70yoa man who presents with a lump in the neck that is clearly not in the thyroid or other midline structures?
A

Thyroid disorders

Non-lymph nodes

  • Lipomas
  • Sebaceous cysts
  • Salivary gland tumours (refer to ENT surgeon)

Lymph nodes

  • Normal
  • Reactive – inflammation or infection e.g. glandular fever, tonsillitis, dental abscess, tuberculosis
  • Malignant e.g. metastatic, lymphoma.

History:

  • Red flags
    • Hoarseness, dysphagia, nasal obstruction or bleeding, oral ulceration, local pain
  • Duration
  • Growth, shrinkage
  • Pain
  • Other lumps?
  • Hx of recent infection?
  • Smoking, alcohol intake
  • Associated symptoms e.g. mouth ulcer, dysphagia, nasal obstruction or bleeding, hoarseness, otalgia
  • Systemic symptoms e.g. weightloss, night sweats,
  • Immigrant from country with high prevalence of tuberculosis.
  • Symptoms of head or neck cancer
    • Dysphonia, stridor, breathing difficulty, cough, haemoptysis, otalgia, unilateral hearing loss, nasal discharge, epitaxis.

Examination

  • Vitals and general inspection
  • Is it superficial or deep?
  • Is it in the anterior or posterior triangle?
  • What is its relationship to muscle?
  • Lump characteristics:
  • Tender or warm?
  • Solid or fluctuant?
  • Pulsatile?
  • Mobile?
  • Examine the mouth and throat
  • Examine all head and neck lymph nodes
  • Examine the skin of the head and neck
  • Palpate the thyroid gland
  • Cranial nerve examination (particularly the facial nerve)
  • Abdominal examination
  • Cardiorespiratory examination

Mass localisation

  • Pre-auricular and angle of jaw: Salivary/ lymphoid tissue in parotid system
  • Central: Thyroid or malignant tissue, dermoid cyst
  • Anterior aspect of SCM: Lymphoid tissue, second branchial cleft cyst
  • Posterior triangle: Lymphoid tissue, likely malignant
  • Supraclavicular: Malignancy from lung/ gynae/ GI source

Characteristics:

  • Reactive: Discrete, mobile, firm/ rubbery, slightly tender
  • Malignant: Rock hard, fixed, matted
  • Infected: Isolated, asymmetric, tender, warm, erythematous, possibly fluctuant
  • Congenital cystic: Soft, ballotable, mobile
  • Carotid body tumour/ vagal schwannoma: Moves side to side, not up and down, firm, lateral mass
  • Vascular lesion: Pulsatile, bruit present
  • Thyroid: Immobile midline, elevates with swallowing à thyroglossal duct cyst, thyroid tumour
123
Q
  1. This man has complained about swelling at his umbilicus.
    1) What can you see and what is the diagnosis?
    2) what is the likely contents?
    3) what is the risk of serious complications with this hernia as it stands.
    4) what would be the correct approach if this were to become acutely painful and incarcerated?
A

1) likely diagnosis: umbilical/ para-umbilical/ incisional hernia
2) Contents

  • Most likely contain peritoneal fat or mesentery
  • May also contain small bowel and/or large bowel
  • May be a Richter’s hernia

3) Risk of serious complications:

  • Incarceration (irreducible)
  • A small neck and irreducible hernia is at risk of strangulation and obstruction, or with a Richter’s hernia there can be strangulation without obstruction
    • Richters hernia: only a portion of the bowel circumference is trapped in the hernial sac. Although the bowel lumen remains patient peristalsis is disrupted -> signs of bowel obstruction

4) approach if this were to become painful and incarcerated:

  • ABCDE
  • Analgesia
  • IV fluids – crystalloids
  • NBM
  • Antiemetics if required
  • Consider NG decompression
  • Consider urinary catheterisation
  • Surgical consult
    • Consider attempting to reduce the hernia
    • Definitive management is open or laparoscopic repair with mesh.
124
Q
  1. This is a laparoscopic view of the lower abdomen
    1) what is visible here?
    2) how can the bowel be injured at laparoscopy and what are the consequences?
A

1) may be appendix, pelvic structures, vessels, bowel, inguinal structures etc.
2) Causes of bowel injury:

  • Thermal injury from electrocautery
  • Mechanical injury from dissection/ manipulation
  • Perforation with placement of trocar/ pneumoperitoneum needle

Consequences:

  • Necrosis
  • Perforation
  • Peritonitis
  • Sepsis
  • Haemorrhage
  • Death